SlideShare une entreprise Scribd logo
1  sur  35
Photobiology and Photosensitivity Disorders
1) What mutation is responsible for this clinical presentation in a patient with thyroid dysfunction
and chronic candidal infections?


A. LYST

B. PTEN

C. AIRECorrect Choice

D. TRP1

E. fumarate hydratase
APECED syndrome named for the clinical features of autoimmune polyendocrinopathy, candidiasis,
ectodermal dystrophy and is caused by a mutation in AIRE (autoimmune regulator). 13% of these
patient have vitigo


2) Treatment of this condition might include:

A. Cyclosporin

B. Antimalarials Correct Choice

C. Cytoxan

D. Azathioprine

E. All of these answers are correct
Sun avoidance, sunblocks, protective clothing, and topical steroids are sufficient for most patients
with PMLE. Other patients may require hardening with UVB or PUVA. Rare patients require
antimalarials


3) Immediate pigment darkening:

A. Is caused by an increase in tyrosinase activity

B. Is predominately brought on by UVA and visible light Correct Choice

C. Is associated with an increase in melanocyte number

D. Start 45-60 minutes after exposure

E. Is predominately brought on by UVB
Immediate pigment darkening appears almost as soon as irradiation occurs. It is due to photo-
oxidation of preexisting melanin


4) A patient present with blue-gray pigmentation on sun-exposed areas but does not have
involvement of the sclerae, lunulae or mucous membranes. Which of the following medications is
could be causing this pigmentation?


A. Sertraline

B. Fluoxetine

C. ChlorpromazineCorrect Choice

D. Clindamycin

E. Ciprofloxacin




                                                     1
Amiodarone, chlorpromazine and tricyclic antidepressants all are capable of causing blue-gray
pigmentation on sun-exposed areas without involvement of the sclerae, lunulae or mucous
membranes


5) UVB acts on what compound to form pre-Vitamin D3 ?

A. 7-deoxycholesterol

B. 9-hydrocholesterol

C. 5-dehydrocholesterol

D. 7-hydrocholesterol Correct Choice

E. DNA
UVB converts 7-dehydrocholesterol in the skin to pre-Vitamin D3, which then thermally isomerizes
to form Vitamin D3


6) Which of the following statements regarding phototesting is correct?

A. To test for a visible light reaction, a slide projector is used as a light sourceCorrect Choice

B. Repeated MEDs to UVB or UVA, given to different test sites over several days can be used to
reproduce lesions of polymorphous light eruption

C. The UVA MED is performed using a narrowband UVA source

D. A single large dose of visible light is the best way to reproduce lesions of polymorphous light
eruption

E. The UVB MED is performed using narrowband UVB light sources
Phototesting is done prior to initiating phototherapy or during provocative induction. To test for a
visible light reaction, a slide projector is used as a light source. The other choices are incorrect. The
UVB MED and UVA MED is performed using BROADBAND UVB and BROADBAND UVA light sources
respectively. Repeated MED's to UVB or UVA given to the SAME site may be used to reproduce
lesions of PMLE. Alternatively a single large dose of UVA or UVB can be used to reproduce PMLE


7) The main contributor to erythema of the skin with exposure to the sun is:

A. UVC

B. Visible light

C. UVA2

D. UVA1

E. UVBCorrect Choice
UVB in natural sunlight is the main contributor to erythema. UVB erythema reaches a maximum in
6-24 hours. UVA accounts for 15-20% of sunlight erythema despite that there is much more UVA
than UVB in sunlight


8) Treatment of CAD might include:

A. All of these answers are correctCorrect Choice

B. PUVA

C. UV filters for car windows




                                                    2
D. Incandescent bulbs for home and office lighting

E. Broad spectrum sunblock to which the patient is not allergic
CAD patients are exquisitely sensitive to UVB, often sensitive to UVA, and sometimes sensitive to
visible light. Answers a, b, and c are correct because those treatments prevent UV and visible
radiation from reaching the patient. PUVA hardens and desensitizes the skin


9) The xenon arc solar simulator, whose spectrum is in this slide, is useful for:

A. MED-B testing and MED-A testing Correct Choice

B. MED-A testing

C. Photopatch testing

D. MED-B testing, MED-A testing, and Photopatch testing

E. MED-B testing
Solar simulator radiation contains both UVB and UVA in sufficient quantities to be useful for MED-B
and MED-A testing. The field size is too small for photopatch testing


10) Rare cases of which photosensitivity disorder have been associated with erythropoietic
protoporphyria?


A. Solar urticariaCorrect Choice

B. Chronic actinic dermatitis

C. Actinic prurigo

D. Hydroa vacciniforme

E. Polymorphous light eruption
Rare cases of solar urticaria have been associated with erythropoietic protoporphyria (EPP), lupus
erythematosus (LE), and with certain drugs. Blood tests for LE and EPP should be performed in
patients with this diagnosis


11) Fluorescent UVA bulbs used for phototesting or PUVA therapy have a peak emission at:

A. 254 nm

B. 468 nm

C. 311 nm

D. 312 nm

E. 352 nmCorrect Choice
254 nm is the wavelength of the radiation emitted by mercury vapor lamps. Narrowband UVB emits
311-312 nm. Fluorescent UVA bulbs used for phototesting or PUVA therapy have a peak emission at
352 nm


12) A patient that rarely burns and usually tans is which of the following skin types?

A. Type IV

B. Type IIICorrect Choice

C. Type I




                                                   3
D. Type V

E. Type II
Skin   type I always burns and never tans
Skin   type II usually burns and rarely tans
Skin   type III rarely burns and usually tans
Skin   type IV never burns and alway tans
Skin   types V,VI are highly pigmented individuals


13) The wavelength range that most effectively induces cutaneous immunosuppression is:

A. 400-410nm

B. 320-340nm

C. 290-320nmCorrect Choice

D. 340-400nm

E. 200-290nm
290-320nm (UVB) is most effective in suppressing cutaneous immunity; mechanisms include
depletion of Langerhans cells, induction of regulatory T cells, and keratinocyte secretion of such
immunosuppressive cytokines as IL-10 and TNF-alph


14) Which of the following statements is true regarding pseudoporphyria?

A. None of these answers are correct

B. It has been associated with furosemide

C. All of these answers are correctCorrect Choice

D. Clinically it may be indistinguishable from porphyria cutanea tarda

E. Porphyrins may be normal
Pseudoporphyria is a phototoxic reaction that clinically and histologically resembles porphyria
cutanea tarda. In pseudoporphyria, however, there is no porphyrin abnormality. It has been
associated with numerous medications, including nalidixic acid, tetracyclines, amiodarone,
furosemide, and ketoprofen


15) Oral PUVA has been used to treat which of the following diseases:

A. All of these answers are correctCorrect Choice

B. Graft-versus-host disease

C. CTCL

D. Mastocytosis

E. Generalized granuloma annulare
PUVA has been reported to have helped patients with all the diseases listed


16) Regarding renal transplant recipients, which of the following statements is correct:

A. Should have skin cancer screening at the same rate as the general population

B. Have a 36-fold increased risk of BCC




                                                     4
C. Have rates of SCC similar to the general population

D. Have an decreased risk for melanoma

E. Have a 36-fold increased risk of SCCCorrect Choice
The rates of SCC in renal cell transplant recipients is increased at a rate of 36x. The rates of skin
cancer are higher than in the general population


17) The usual dose of UVR given for photopatch testing is approximately:

A. 10 Joules of UVA

B. 4 millijoules of UVB

C. 4 Joules of UVB

D. 2 Joules of UVA Correct Choice

E. 10 Joules of UVB
UVA is absorbed by the chemicals that cause photoallergic contact dermatitis; therefore, it is UVA
that is used for photopatch testing. In patients who are UVA-sensitive, one-half of their MEDA is
used for photopatch testing


18) A patient presents with signs of porphyria cutanea tarda. Porphyrin screens are negative. Which
of the following medications on the patient's medication list is your top choice for discontinuation?


A. Fenofibrate

B. Multivitamin

C. NaproxenCorrect Choice

D. Chloroquine

E. Enalapril
NSAIDs are a frequent offender in causing pseudoporphyria. The other listed medications are not
frequent causes of this type of skin reaction


19) The typical patients with polymorphous light eruption are:

A. Fair-skinned females in their teens and 20’s. Correct Choice

B. Type V-skinned females in their 20’s and 30’s.

C. Fair-skinned males in their 20’s and 30’s.

D. Fair-skinned females in their 40’s and 50’s.

E. Type IV-skinned males in their 40’s and 50’s.
PMLE most commonly appears in fair-skinned females during the first three decades of life. It may
be related to type IV hypersensitivity


20) All of the following statements regarding chronic actinic dermatitis are true EXCEPT:

A. Azathioprine has been used successfully to treat this disorder

B. The MEDB on phototesting is markedly diminished in patients with this condition

C. Many cases begin as photoallergic contact dermatitis or drug photosensitivity




                                                    5
D. Fluorescent bulbs are safer for these patients than are incandescent bulbsCorrect Choice

E. Many patients have a lowered threshold to shorter wavelength visible light
Chronic actinic dermatitis (CAD) usually occurs in middle-aged to elderly males who present with a
chronic, eczematous dermatitis in a photodistribution, though there is no history of current
exposure to a photosensitizer. Phototesting is very helpful in diagnosing CAD. The MEDB is
markedly diminished, and the MEDB site may show an eczematous or infiltrated appearance. Many
of the patients have a lowered MEDA as well, and may have a lowered threshold to shorter
wavelength visible light in the blue-violet end of the spectrum. Many cases of this idiopathic
disorder are thought to have begun as photoallergic contact dermatitis or as a drug photosensitivity
with broadening of the photosensitivity to include the UVB range. It is unclear why photosensitivity
persists when the photosensitizer is no longer present. Treatment includes strict sun avoidance and
sun protection. Incandescent bulbs with longer wavelengths, far from the blue-violet end of the
visible spectrum, should be used instead of fluorescent bulbs, which have significant blue-violet
radiation. Topical and oral steroids, oral azathioprine and cyclosporine, and PUVA have all been used
to treat patients with CAD


21) Treatment of polymorphous light eruption includes all of the following EXCEPT:

A. None of these answers are correctCorrect Choice

B. PUVA therapy

C. Systemic corticosteroids

D. Antimalarials

E. Topical steroids
Most patients with PMLE have mild disease that can be treated by sun avoidance and sun
protection. Topical steroids can be used to treat clinical lesions. For severe cases, hardening and
desensitization can be accomplished with UVB, UVB plus UVA, or PUVA. Antimalarials can be used
for resistant cases. A short course of prednisone (20-40 mg) is effective for brief, sunny vacations


22) Which of the following statements about ultraviolet C is incorrect?

A. UV-C is absorbed by atmospheric ozone

B. UV-C has an electromagnetic spectrum from 200-290 nm

C. UV-C has a higher wavelength than UV-BCorrect Choice

D. UV-C has higher energy than UV-B

E. UV-C does not reach the earth's surface
UV-C has wavelengths of 200 - 290 nm. UV-B has wavelengths of 290 - 320 nm. UV-C has a lower
wavelength, not higher. All of the other listed statements about UV-C are correct


23) Which of the following statements about the spectrums of UVR that cause of solar urticaria is
MOST correct?


A. Visible light causes solar urticaria

B. UVA causes solar urticaria

C. UVB causes solar urticaria

D. Both UVA and UVB cause solar urticaria

E. Patients can react to visible light, UVA and/or UVBCorrect Choice




                                                  6
Some patients react with wheals to either visible light or UVA or UVB. Others react to both UVA and
visible radiation. Some react to both UVB and UVA, and some patients react to UVB, UVA and visible
light


24) Patients with chronic actinic dermatitis (CAD) typically show on phototesting:

A. Lowered MEDB, usually normal MEDA, sometimes positive photopatch tests

B. Lowered MEDB, usually lowered MEDA, sometimes positive photopatch tests Correct Choice

C. Normal MEDB, lowered MEDA, sometimes positive photopatch tests

D. Normal MEDB, normal MEDA, sometimes positive photopatch tests

E. None of the above
The hallmark of CAD is a lowered MEDB. Often the patients have a lowered MEDA. It is felt that
many of CAD patients began with photoallergic contact dermatitis, so some have positive
photopatch tests


25) Most patients with PMLE require treatment with:

A. Sunscreen and sun avoidance between 11 am and 3pmCorrect Choice

B. Cyclophosphamide

C. Chloroquine

D. UVB hardening/desensitization

E. Prednisone
Most patients have mild disease that can be treated by sun avoidance, especially between 11 am
and 3pm; a broad spectum sunscreen and clothing with a tight weave. In more severe cases, UV
hardening, antimalarials or prednisone can be used. Cyclophosphamide is not used in PMLE


26) The histology of the patient in question 42 is the following slide. It mimics the histology of:

A. CTCL Correct Choice

B. Pityriasis rosea

C. Psoriasis

D. Tuberculoid leprosy

E. SLE
The histology of the actinic reticuloid variety of CAD mimics that of mycosis fungoides with a band-
like infiltrate with atypical mononuclear cells


27) The highest energy visible photons are in which portion of the visible spectrum?

A. All of these answers are correct

B. Green

C. Yellow

D. Blue-violet Correct Choice

E. Red-Orange




                                                   7
In the visible spectrum, the blue-violet portion has the shortest wavelength and the highest energy.
The red-orange portion has the longest wavelength and the lowest energy


28) In solar urticaria wheals typically:

A. Begin at 1-2 h. and last 2-4 h

B. Begin at 15-30 minutes and last 1 h Correct Choice

C. Begin at 45 minutes after exposure and last 2 hours (h.)

D. Begin at 15-30 seconds and last about 15 minutes

E. Begin at 30-60 minutes and last 1-4 h
The wheals of solar urticaria begin 10-15 minutes after exposure and last for about an hour. Solar
urticaria is usually idiopathic. Rare cases are associated with EPP, SLE, and certain medications


29) Currently the most common cause of photoallergic contact dermatitis is:

A. Musk ambrette

B. Halogenated salicylanilides

C. Mercaptobenzothiazole

D. 6-methylcoumarin

E. Sunscreens Correct Choice
Halogenated salicylanilides, musck ambrette and 6-methylcoumarin used to cause most of the cases
of photoallergic contact dermatitis. They have been essentially eliminated from soaps and
fragrances. Sunscreen ingredients are now the most common cause


30) The potent photosensitizer, 5-methoxypsoralen, is contained in which of the following contact
allergens?


A. Eugenol

B. Oil of BergamotCorrect Choice

C. Usnic acid

D. Balsam of Peru

E. Tuliposide A
Oil of Bergamot contains 5-methoxypsoralen and may cause of 'Berloque dermatitis,' an intense
post-inflammatory hyperpigmentation due to an antecedent phytophotodermatitis that may be
subclinical


31) A common cause of medication induced photoallergy is:

A. Naproxen

B. Acitretin

C. Psoralens

D. PiroxicamCorrect Choice

E. Ibuprofen




                                                 8
All of the listed medications are causes of photosensitivity, but only piroxicam is a cause of
photoallergy


32) Phototesting of PMLE patients reveals:

A. Normal MEDB, reduced MEDA.

B. Normal MEDB, elevated MEDA.

C. Normal MEDB, normal MEDA. Correct Choice

D. Lowered MEDB, normal MEDA.

E. Lowered MEDB, lowered MEDA.
The MEDB and MEDA are normal in PMLE patients. Only with multiples of the MEDB or MEDA can
one often reproduce the lesions


33) Ultraviolet radiation from the sun cause which of the following acute effects EXCEPT?

A. Increased immune surveillance Correct Choice

B. Immediate pigment darkening

C. Epidermal thickening

D. Sunburning

E. Photosynthesis of vitamin D
UV radiation causes acute effects including: photosynthesis of vitamin D, sunburning, immediate
pigment darkening, delayed tanning, epidermal thickening and immunologic effects such as
DECREASED immune surveillance


34) A 40-year-old patient presents with widespread plaque-type psoriasis without arthritis. She had
already applied potent topical steroids, calcipotriene, and tazarotene. The treatment of choice would
be:


A. Methotrexate

B. Narrowband UVB Correct Choice

C. Etretinate

D. Cyclosporine

E. PUVA
The other choices involve internal medications and therefore have the potential for more side effects
than with narrowband UVB. Phototherapy may be impractical for some patients, and therefore
systemic therapies should be offered


35) Which of the following is true regarding actinic prurigo?

A. Lesions generally continue through late adulthood

B. Cheilitis is frequently seenCorrect Choice

C. Thalidomide has been ineffective for the majority of patients

D. Lesions persist for 1-2 days

E. Lesions never occur on non-sun-exposed areas




                                                   9
Actinic prurigo is an idiopathic photosensitivity disorder. Lesions are excoriated papules and nodules
that begin in childhood and remit in puberty. They can last for several months and may occur on
non-sun-exposed areas. Thalidomide has been very effective for treating the majority of patients
with actinic prurigo.


36) The irradiance of a UV source is measured in:

A. Joules

B. Seconds

C. Millijoules

D. Centimeters

E. Watts Correct Choice
The irradiance of a UV source in watts x the time in seconds equals the UV dose in joules


37) A patient presents with solar urticaria. What tests should be considered initially?

A. ESR

B. ANACorrect Choice

C. CXR

D. ANCA

E. Urinalysis
Solar urticaria is an idiopathic, type I photosensitivity disorder. Rare cases have been associated
with erythropoietic protoporphyria (EPP) and lupus erythematosus (LE). Blood tests for LE and
appropriate screening for EPP should be performed including ANA, Ro/La, Urine & Stool porphyrins


38) The best location for phototesting patients suspected of photosensitivity is:

A. Unaffected skin of the upper back

B. Effected skin of the ventral forearm

C. Effected skin of the buttock

D. Unaffected skin of the lower backCorrect Choice

E. Unaffected skin of the outer thighs
Patients with suspected photosensitivity can be tested on unaffected skin of the buttocks, lower
back or ventral forearm. Effected skin should not be used for testing


39) Blue-gray pigmentation of the lunulae:

A. Is a rare side effect of treatment with tricyclic antidepressants

B. None of these answers are correct

C. Occurs only after many years of treatment with chlorpromazine

D. Is seen in argyria and not in most medication photosensitivitiesCorrect Choice

E. Can be seen in patients on amiodarone
Blue-gray pigmentation on sun-exposed areas can be seen during treatment with amiodarone,
chlorpromazine, and tricyclic antidepressants. In these medication photosensitivities, there is no




                                                   10
involvement of the sclera, lunulae, or mucous membranes, as in argyria (prolonged contact with or
ingestion of silver salts


40) The best definition for a MED is:

A. The dose of ultraviolet radiation that produces pronounced erythema that completely fills the test
square

B. The dose of ultraviolet radiation that is one step below the first visible erythema

C. The dose of ultraviolet radiation that produces bullae that completely fills the test square

D. The dose of ultraviolet radiation that produces a easily visible "sunburn"

E. The dose of ultraviolet radiation that produces barely perceptible erythema that completely fills
the test squareCorrect Choice
An MED is the dose of ultraviolet radiation that produces barely perceptible erythema that
completely fills the test square. The remaining options are incorrect


41) The image shown is diagnostic of which of the following conditions?

A. Bullous pemphigoid

B. Pemphigus vulgaris

C. Chronic actinic dermatitis

D. Polymorphous light eruptionCorrect Choice

E. Pseudoporphria cutanea tarda
The pathologic image shown is that of polymorphous light eruption. The histology shows a
lymphocytic infiltrate around the superficial vascular plexuses with subepidermal edema and
minimal epidermal changes are present


42) Photoexacerbated genodermatoses include all except:

A. Hartnup Disease

B. Darier's Disease

C. Hailey-Hailey Disease

D. Cockayne's Syndrome

E. Tuberous sclerosis Correct Choice
Tuberous sclerosis is not associated with photosensitivity


43) Which of the following statements is correct regarding antigen presenting cells after UVR
exposure?


A. Have increased ability to prime UV-irradiated mice to subcutaneously injected hapten

B. UV-irradiated mice have normal antigen presentation, allowing a normal delayed-type
hypersensitivity response

C. APC's have increased ability to prime UV-irradiated mice to subcutaneously injected protein

D. APC's have increased ability to prime UV-irradiated mice to applied contact-sensitizing agents

E. There is a decrease in the number of antigen presenting cellsCorrect Choice




                                                   11
APC's have depressed ability to prime UV-irradiated mice to subcutaneously injected hapten or
protein and to applied contact-sensitizing agents. UV-irradiated mice have defective antigen
presentation, preventing a normal delayed-type hypersensitivity response. There is a REDUCTION in
number of antigen presenting cells


44) The UVAII portion of the electromagnetic spectrum extends from:

A. 290-320 nm

B. 320-340 nmCorrect Choice

C. 320-400 nm

D. None of these answers are correct

E. 200-290 nm
UVA extends from 320-400nm. UVA is further subdivided into UVAII (320-340nm) and UVA1
(340-400nm


45) Which of the following statements about UVR and DNA is correct?

A. Pyrimidine dimers can not activate oncogenes

B. UVA is most effective in producing pyrimidine dimer

C. UVA is much more efficient than UVB in inducing DNA damage

D. Cells from patients with actinic keratoses have more DNA repair capacity than controls

E. UVR alters DNACorrect Choice
UVR alters DNA. UVB is much more efficient than UVA in inducing DNA damage. It can create
pyrimidine dimers that may activate oncogenes and lead to cancer. Cells from patients with actinic
keratoses have less DNA repair capacity than controls


46) Ultraviolet light spectrum most completely encompasses which of the following spectrums?

A. 760 - 1200 nm

B. 290 - 400 nm

C. 10 - 400 nmCorrect Choice

D. 400 - 700 nm

E. 200 - 400 nm
The ultraviolet spectrum encompasses Vacuum UV from 10-200nm, UV-C from 200-290nm, UV-B
from 290-320nm and UV-A from 320-400nm. The visible light spectrum is from 400-700nm.
700-1200 nm is part of the infrared spectrum


47) Which of the following is true about UV light:

A. Clouds absorb most UVA light

B. Sunlight early in the morning and late in the day contains relatively more UVB

C. UVB radiation is 1000 times more erythrogenic than UVACorrect Choice

D. UVAII light is 340-400nm

E. UVA radiation is 1000 times greater than UVB during midday hours




                                                  12
UVA light is found b/w 320 and 400nm and is broken up into UVAI(340-400nm) and
UVAII((320-340nm). UVB light is found between 290 and 320nm. UVA radiation is 100 times
greater than UVB during midday hours and sunlight early in the morning and late in the day
contains relatively more UVA. UVB radiation is 1000 times more erythrogenic than UVA. Cloud cover
is a poor UV absorber


48) Which of the following hypoglycemics is the most common cause of photosensitivity?

A. Thiazolidinediones (i.e. rosiglitazone)

B. Metformin

C. Insulin

D. Piroxicam

E. SulfonylureasCorrect Choice
The sulfonylurea hypoglycemics for diabetes are the most common type of medication of this class
of medications. Piroxicam is not a hypoglycemic agent, but a common NSAID cause of photoallergy


49) The most helpful phototest to document this photosensitivity disorder would be:

A. MEDBB

B. Repeated doses of UVA and UVB Correct Choice

C. Photopatch tests

D. MEDNB

E. All of these answers are correct
Repeated doses of ultraviolet radiation can sometimes elicit lesions of PMLE. More patients react to
UVA radiation than to UVB radiation


50) Hereditary PMLE of Native Americans:

A. Often presents with chelitis and conjunctivitisCorrect Choice

B. Rarely persists into adulthood

C. All of these answers are correct

D. Is not treated with thalidomide

E. Is more similar to PMLE than to actinic prurigo
Hereditary PMLE of Native Americans is similar to actinic prurigo but persists much more frequently
into adulthood. 75% of patients have a positive family history. It presents with a papular,
excoriated, eczematous dermatitis that occurs predominantly on the face. Chelitis and conjunctivitis
are common. It may be treated with thalidomide


51) Solar urticaria:

A. Rarely lasts for more than 6 months to 1 year

B. Is not benefitted by antihistamines

C. Can present with headache, nausea, and syncopeCorrect Choice

D. Is an idiopathic, type IV photosensitivity disorder




                                                     13
E. Usually occurs to only UVB radiation
Solar urticaria is an idiopathic, type I photosensitivity disorder. Mediator release during widespread
whealing may result in headache, nausea, wheezing, faintness, and syncope. It usually lasts for
many years. Some patients react with wheals to either visible light or UVA or UVB. Others react to
both UVA and visible radiation. Some react to both UVB and UVA, and some patients react to UVB,
UVA, and visible radiation. Treatment includes sun avoidance and protection, and H-1
antihistamines may be of partial benefit


52) Most bulbs used for PUVA have a peak output predominantly in the following range:

A. 320 nm – 340 nm

B. 390 nm – 410 nm

C. 340 nm – 400 nm

D. 350 nm – 360 nm Correct Choice

E. 290 nm – 320 nm
The UVA emitted by these bulbs is absorbed by psoralens, causing covalent bonding of psoralens to
DNA


53) The active spectrum for cutaneous vitamin D3 synthesis is:

A. 320-400nm

B. 400-410nm

C. 290-320nmCorrect Choice

D. 410-450nm

E. 220-290nm
The source states that vitamin D3 synthesis occurs at wavelengths < 320 making choices
220-290nm and 290-320nm correct


54) When solar urticaria is a consideration for phototesting:

A. 7 test squares of increasing UV doses should be exposed

B. An MED (B) and MED (A) should not be performed

C. The lower back should not be used for testing

D. Visible light will not evoke the lesions

E. An additional reading at 15 minutes after exposure should be performedCorrect Choice
When solar urticaria is a consideration, an additional reading at 15 minutes after exposure is
important, as wheals begin within 10-30 minutes after exposure and last for about one hour. The
face and hands may not show lesions as they are chronically exposed to sun. Some patients react to
either visible light or UVA or UVB. Others react to both UVB and UVA, both UVA and visible light, or
all three


55) Which of the following statements regarding hereditary PMLE of Native Americans is true?

A. Specific HLA types predominate in Native AmericansCorrect Choice

B. Cheilits and conjunctivitis are uncommon




                                                   14
C. 5-10% of patients have a family history of this eruption

D. It will remit in puberty

E. Patients have an urticarial erupion.
Native Americans have a high rate of PMLE and there is some evidence of a genetic predisposition.
It tends to have a specific HLA predominance, continues through adulthood, and commonly presents
as a papular, excoriated, eczematous dermatitis predominantly on the face. Cheilitis and
conjunctivitis are common. Up to 75% of patients have a positive family history


56) The portion of the electromagnetic spectrum that produces a particular biologic effect is known
as the:


A. Photobiologic spectrum

B. Minimal erythema dose

C. Active spectrum

D. Action spectrumCorrect Choice

E. Absorption spectrum
The action spectrum is the portion of the electromagnetic spectrum that products a particular
biologic effect (e.g. erythema, delayed tanning). The absorption spectrum is the portion of the
electromagnetic spectrum that is absorbed by a particular absorbing molecule, or chromophore. The
minimal erythema dose is the dose of UV radiatino that produces barely perceptible erythema
during phototesting


57) A patient presents with onycholysis after sun exposure. Which of the following medications
would you rule out as a cause of this presentation?


A. Quinolones

B. Quinine

C. ChlorpromazineCorrect Choice

D. Tetracyclines

E. Psoralens
Chlorpromazine is associated with blue-gray pigmentation on sun-exposed areas and is not
associated with photoonycholysis. Quinolones, tetracyclines, psoralens and quinine can cause
photoonycholysis


58) Phytophotodermatitis can be seen with the following fruits / vegetables:

A. Potatoes and leeks

B. Tomatoes and bananas

C. Celery and radishes

D. Parsnips and limesCorrect Choice

E. Lemons and pears
Psoralens in certain plants, fruits, and vegetables can cause phytophotodermatitis. The most
common ones are limes, figs, parsley, parsnip, bergamot oranges, and celery


59) Narrow band UVB consists of what wavelength?



                                                 15
A. 308-310nm

B. 311-313nmCorrect Choice

C. 300-306nm

D. 312-320nm

E. 320-330nm
Narrowband UVB is much less erythemogenic with regard to physical units (mJ/cm2) than
broadband UVB. Narrowband UVB is 311-313nm


60) Absorption of UV radiation generates singlet oxygen in the skin by which chromophore?

A. Melanin

B. Water

C. Hemoglobin

D. Keratin

E. Urocanic acidCorrect Choice
Urocanic acid and DNA are biologically important chromophores. DNA absocrbs uVB directly inducing
changes between adjacent pyrimidine bases on one strand of DNA. Cyclopyrimidine dimers,
particularly thymine dimers or less commonly (6-4) photoproducts may be generated. Urocanic acid
is a second biologically important chromophore in the skin and is a by-product of filaggrin
breakdown. One photon of light contains enough energy to generate singlet oxygen.


61) Which of the following cell types induce susceptibility to tumor growth?

A. NK cells

B. Langerhans cells

C. Helper T-cells

D. Suppressor T-cellsCorrect Choice

E. Mast cells
The suppressor T-cells induce susceptibility to tumor. These cells appear to arise in UV-irradiated
hosts prior to tumor developing, and play a role in carcinogenesis


62) Possible treatments for this patient with vitiligo include all of the following except:

A. Topical steroids

B. AcitretinCorrect Choice

C. Narrowband UVB

D. Oral PUVA

E. Broadband UVB
Topical steroids and all the UV choices have been used to treat vitiligo. Soriatane does not benefit
vitiligo


63) The solar simulator is not useful for phototherapy because of its:

A. Low output of UVB



                                                   16
B. High output of infrared radiation

C. Small field sizeCorrect Choice

D. Low level of ionizing radiation

E. High output of ultraviolet radiation
The coin-sized field of the solar simulator prevents this source from being useful for phototherapy.
It is useful in MED testing


64) All of the following are true regarding actinic reticuloid except:

A. Is a premalignant conditionCorrect Choice

B. CD8+ T cells in lesional skin

C. Generalized lymphadenopathy common

D. Atypical dermal mononuclear cell infiltrate

E. Affects elderly men
Actinic reticuloid is a type of chronic actinic dermatitis. Ive et al. introduced the disease as a severe
dermatosis with no apparent photoallergen. It generally affects elderly males and is characterized
by infiltrated erythematous plaques on an eczematous background in exposed sites with
lymphadenopathy. Histopathologically, it may resemble cutaneous T cell lymphoma. However, there
is a trend towards a lower CD4+/CD8+ ratio. It is not considered a premalignant condition


65) Which medication reactivates UVB- and PUVA-induced erythema?

A. MethotrexateCorrect Choice

B. 5-FU

C. None of these answers are correct

D. Dacarbazine

E. Vinblastine
Methotrexate reactivates UVB- and PUVA-induced erythema


66) The patient demonstrates a positive photopatch test to musk ambrette and a lowered MEDB.
The correct diagnosis is most likely:


A. Solar urticaria

B. Photoallergic contact dermatitis

C. CAD Correct Choice

D. Actinic prurigo

E. PMLE
Only chronic actinic dermatitis (CAD) has a lowered MEDB. That finding helps distinguish CAD from
photoallergic contact dermatitis in which one sees a positive photopatch test also


67) A joule is a measurement of:

A. Wavelength

B. Power



                                                   17
C. EnergyCorrect Choice

D. Irradiance

E. Distance
A joule is a measurement of energy dose or fluence. Power or irradiance is measured in watts.
These are related by the formula Joules/cm2=Watts/cm2xseconds


68) What is the most likely cause of these lesions on the cheek of this 8 year-old boy?

A. Varicella

B. Herpes simplex

C. Polymorphous light eruption

D. Acne

E. Hydroa vacciniformeCorrect Choice
Hydroa vacciniforme is a rare photodermatosis of childhood which occurs on areas of sun-exposed
skin. The lesions leave depressed and atrophic scars. The condition tends to spontaneously resolve
in adulthood


69) Lesions of PMLE typically appear:

A. Immediately

B. Hours to days after exposureCorrect Choice

C. About one hour after exposure

D. Days to two weeks after exposure

E. 15-30 minutes after exposure
The history of a delay of several hours to several days after exposure is important to the diagnosis


70) The following compound exerts immunosuppressive effects in the skin following exposure to
UV-radiation:


A. Cis-urocanic acidCorrect Choice

B. Delta aminolevulinic acid

C. IL-12

D. Amino-levulinic acid

E. 7-dehydrocholesterol
Trans-urocanic acid is an epidermal chromophore that isomerizes to cis-urocanic acid following
exposure to UV radiation. Cis-urocanic acid has been shown to be immunosuppressive, for example,
by impairing the induction of contact allergy in mouse models. The mechanism of this
immunosuppressive effect is unclear


71) UVA II encompasses which wavelengths ?

A. 290-320 nm

B. 340-400 nm

C. 400-450 nm



                                                 18
D. 320-340 nm Correct Choice

E. 320-400 nm
UVA can be divided into UVA II (320-340 nm) and UVA I (340-400 nm)


72) In solar urticaria wheals may be brought on by:

A. UVA, UVB and Visible light Correct Choice

B. UVB

C. UVC

D. UVA

E. Visible light
Some patients react to visible light or UVA or UVB. Others react to combinations of visible light,
UVA, and UVB


73) All of the following are true about UVA radiation except:

A. penetrates to a greater depth in the dermis than UVB

B. virtually all blocked by car window glassCorrect Choice

C. responsible for phototoxic drug reactions

D. 10 times more abundant than UVB

E. approximately 50% of exposure occurs in the shade
The UVA band extends from 320 to 400 nm. This spectrum is further subdivided into UVA-2 (320 to
340 nm) and UVA-1 (340 to 400 nm). The UVA spectrum is recognized as a cause of immediate and
delayed tanning reaction of skin, and several other effects including photoaging, skin
photosensitization, and immunosuppression. The majority of the ultraviolet radiation at the earth's
surface is UVA (95 to 98%) with only 2 to 5% comprised of UVB. As UVC is completely absorbed by
the stratospheric ozone layer, it does not comprise ultraviolet radiation hitting the earth's surface.
Much of the UV radiation after reaching the atmosphere becomes scattered by the time it hits the
earth's surface. Due to this "sky radiation", it is possible to sunburn even if one is exposed only to
the shade. Notably, window glass filters out ultraviolet wavelengths shorter than 320 nm, so only
UVB (290 to 320 nm) and UVC (200 to 290 nm) are effectively filtered by car window glass.
Although UVA penetrates deeper into the dermis than UVB, UVB radiation is much more
erythmogenic. Finally, most common photosensitizers have action spectrums in the UVA range, and,
as a result, UVA radiation is responsible for most phototoxic drug reactions


74) This malnourished individual presented with crusting and hyperpigmentation in a
photodistribution. The best diagnosis is:


A. PMLE

B. Photoallergic contact dermatitis

C. Scurvy

D. CAD

E. Pellagra Correct Choice
Pellagra is characterized by the triad of diarrhea, dermatitis, and dementia. The dermatitis begins as
a burning erythema in sun-exposed areas. There may be bullae and erosions. This is followed by a
dry, brittle, scaling and hyperpigmented phase. Pellagra is due to a deficiency of niacin and
tryptophan




                                                  19
75) Medication photosensitivity is caused by all except:

A. Cephalosporins Correct Choice

B. Thiazides

C. Phenothiazenes

D. Doxycycline

E. Quinolones
Cephalosporins do not cause drug photosensitivity. Neither do the penicillins


76) Which of the following statements about electromagnetic radiation is MOST correct?

A. Electromagnetic radiation is measured in watts

B. The energy of photons is proportional to the wavelength

C. The energy of photons is inversely proportional to the frequency

D. Longer wavelengths penetrate the skin more deeplyCorrect Choice

E. Electromagnetic radiation can be conceptualized as packets of power called photons
Longer wavelengths penetrate the skin more deeply. Electromagnetic radiation can be
conceptualized as packets of ENERGY called photons. The energy of photons is proportional to the
FREQUENCY and inversely proportional to WAVELENGTH. Electromagnetic radiation is measured in
wavelength


77) A MED phototest should be read at:

A. 24 hoursCorrect Choice

B. 12 hours

C. 48 hours

D. 96 hours

E. 2 hours
MED testing should be read 24 hours after delivery of the doses. An additional reading at 15
minutes is important when solar urticaria is a consideration


78) Lichen planus-like lesions on sun-exposed areas may be seen in patients being treated with:

A. None of these answers are correct

B. All of these answers are correct

C. Quinolones

D. Ketoprofen

E. FenofibrateCorrect Choice
Lichenoid medication photosensitivity, with LP-like lesions on sun-exposed areas has been
associated with treatment with antimalarials, thiazides, demethylchlortetracycline, fenofibrate,
enalapril, quinine, and quinidine


79) Photoonycholysis has been attributed to:


                                                 20
A. Tricyclic antidepressants

B. QuinineCorrect Choice

C. Furosemide

D. Chlorpromazine

E. Amiodarone
Photoonycholysis is a manifestation of medication photosensitivity that has been attributed to
quinolones, tetracyclines, psoralens, and quinine


80) The immunologic effects of UVR include all of the following except:

A. Decrease in IL-1, IL-6, TNF-á Correct Choice

B. Alteration of lymphocyte population

C. Suppression of delayed-type hypersensitivity

D. Alteration of Langerhans cell function

E. Alteration of Langerhans cell morphology
UVR causes an increase of circulating cytokines (IL-1, IL-6, TNF-a).


81) Regarding the UVR effects on contact dermatitis and delayed-type hypersensitivity, which of the
following statements is correct?


A. There are increases in production of Th2 type cytokines

B. There are increased delayed-type hypersensitivity responses

C. Induction of sensitization is increased

D. Mice exposed to long-term, high-dose UVR demonstrate increased splenic APC function

E. There are diminished contact hypersensitivity responsesCorrect Choice
Mice exposed to short-term, high-dose UVR demonstrate decreased splenic APC function. There are
diminished delayed-type hypersensitivity and contact hypersensitivity responses. Induction of
sensitization is decreased. Th2 cytokines are not increased following UVR exposure


82) This patient’s MEDB was less than 1 mJ/cm2. His medications were Lipitor, digoxin, and
Coumadin. The most likely diagnosis is:


A. Photoallergic contact dermatitis

B. SLE

C. CAD Correct Choice

D. PMLE tested

E. Drug photosensitivity
CAD is the most likely cause of such a low MEDB. Drug photosensitivity can show a lowered MEDA
and on occasion a lowered MEDB, but none of the patient’s drugs cause photosensitivity


83) Phototoxic reactions:

A. Rarely occur on the first exposure to the chemical




                                                  21
B. Resolve with hyperpigmentationCorrect Choice

C. Occur only in predisposed individuals

D. Are immunologically mediated

E. Are called "photoreactive" if they produce damage through reactive oxygen species
A phototoxic reaction appears as a exaggerated sunburn with erythema and sometimes blistering,
resolving with hyperpigmentation. It is a nonimmunologic reaction that could occur in all individuals
given enough of the chemical and enough UVR. It can occur on the first exposure to the chemical
and the UVR. Phototoxic reactions that produce damage through reactive oxygen species are called
"photodynamic."


84) Initial treatment of this disease should include:

A. Hydroxychloroquine

B. Sun avoidance, sunblocks, desensitization

C. Car UV filters, PUVA, sunblocks

D. Sun avoidance, sunblocks, beta carotene

E. Sun avoidance, sunblock, antihistamines Correct Choice
Treatment of solar urticaria is difficult. Sun avoidance is the most important aspect of treatment. A
broad-spectrum sunblock and antihistamines


85) UVB converts 7-dehydrocholesterol in the skin to:

A. Previtamin D3Correct Choice

B. Calcitriol

C. 1,25-dihydroxyvitamin D

D. None of the above

E. 25-hydroxyvitamin D
UVB converts 7-dehydrocholesterol in the skin to previtamin D3, which then thermally isomerizes to
form vitamin D3. It is hydroxylated in the liver and then in the kidney to form 25-hydroxyvitamin D
and 1,25-dihydroxyvitamin D, respectively


86) Lichen planus like lesions on sun-exposed areas may be seen in patients receiving which
medication?


A. Alprazolam

B. Griseofulvin

C. FenofibrateCorrect Choice

D. All of these answers are correct

E. None of these answers are correct
LP-like lesions (which may be confluent) on sun-exposed areas have been seen in patients receiving
antimalarials, thiazides, demethylchlortetracycline, fenofibrate, enalapril, quinine, and quinidine


87) Common side effects of PUVA include all of the following except:




                                                  22
A. Painful erythema

B. Squamous cell carcinoma

C. Hair loss Correct Choice

D. Prolonged pruritus

E. Nausea
Alopecia is not a usual side effect of PUVA treatment


88) Which of the following statements about the light sources for phototesting/phototherapy is
correct?


A. The mercury vapor is excited by electric current and emits a line spectrum of 254 nmCorrect
Choice

B. Phototherapy bulbs are low-pressure sulfur vapor lamps with the inner surface coated by a
specific phosphor

C. The phosphor emits a discoherent spectrum of various wavelengths

D. The most common light sources are incandescent bulbs

E. Broadband UVB bulbs emit throughout the UVB range and also include some UVC
Fluorescent bulbs are commonly used for phototherapy. These bulbs are low-pressure mercury
vapor lamps with the inner surface coated by a specific phosphor. The phosphor emits a continuous
spectrum of various wavelengths. The mercury vapor is excited by electric current and emits a line
spectrum of 254 nm. Broadband UVB bulbs emit throughout the UVB range and also include some
UVA, not UVC.


89) Lumisterol is an inactive epidermal reservoir of which vitamin?

A. E

B. K

C. A

D. DCorrect Choice

E. C
Lumisterol and tachysterol are inert byproducts created during the biosynthesis of vitamin D. When
Previtamin D3 is exposed to light, it can result in photoisomerization of previtamin D3 to lumisterol
and tachysterol. If previatmin D3 is depleted, lumisterol and tachysterol can become converted back
to previtamin D3


90) Oxsoralen plus UVA results in the following except:

A. Has immunomodulating effects

B. Suppresses DNA synthesis

C. Forms monofunctional adducts

D. Binds to purine bases Correct Choice

E. Can form DNA crosslinks
Oxsoralen, in the presence of UVA, forms covalent bonds to pyrimidine bases on DNA




                                                 23
91) Advantages of narrowband UVB over PUVA therapy are the following except:

A. No nausea

B. Safe in pregnancy

C. No need for protective eyewear

D. Safe in childhood

E. More effective in treating thick plaques of CTCL Correct Choice
UVB, both narrowband and broadband, is less penetrating into the skin than UVA. Therefore, PUVA
is more effective for thick plaques of CTCL. The UVB does not reach to the bottom of the plaques


92) This middle-aged man demonstrates infiltrated, dusky plaques on all sun-exposed areas. The
most likely diagnosis is:


A. Actinic prurigo

B. CAD Correct Choice

C. Drug photosensitivity

D. Photoallergic contact dermatitis

E. PMLE
The thick, infiltrated plaques on sun-exposed areas are typical of the actinic reticuloid variety of
CAD


93) Which of the following statements about the hypothetical effect of UVR on cancer induction is
correct?


A. UVR alters APC function, by increasing the number of antigen-presenting cells

B. Suppressor T-cells are suppressed

C. UVR induces transformation of keratinocytes with expression of tumor-associated antigensCorrect
Choice

D. UVR increases the ability of Langerhans cells to present antigen

E. UVR inhibits the release of immunosuppressive factors
UVR induces transformation of keratinocytes with expression of tumor associated antigens. It alters
APC function by reducing the number of antigen presenting cells. It also promotes the release of
immunosuppressive factors, induce suppressor t-cells, and decreases the ability of Langerhans cells
to present antigen


94) A watt is a measurement of:

A. PowerCorrect Choice

B. Fluence

C. Energy

D. Distance

E. Irradiance
A watt is a measurement of power or irradiance of a UV source. Fluence and energy is measured in
joules. These are related by the formula Joules/cm2=Watts/cm2xseconds




                                                   24
95) A patient presents with erythematous pruritic papules on exposed areas that appear in the
spring. They appear between 2 hours and 2 days after exposure. Which of the following statements
is NOT correct?


A. The pathogenesis is unclear, but may be related to type IV hypersensitivity reactions

B. It is more common in fair-skinned females

C. Vesicles and eczematous dermatitis is uncommon

D. The diagnosis described above is solar urticariaCorrect Choice

E. This is an idiopathic disease that appears in the first three decades of life
The diagnosis described above is polymorphous light eruption. The time to development of lesions is
important in distinguishing between PMLE and solar urticaria. Solar urticaria usually develops 10-30
minutes after UVR exposure


96) Ultraviolet radiation from the sun causes all of the following acute effects in the skin EXCEPT:

A. Photooxidation of preexisting melanin

B. Redistribution of melanosomes from a perinuclear position into dendrites

C. Epidermal thickening

D. Mast cell degranulation

E. None of these answers are correct Correct Choice
All of these statements are acute effects of UV raditation on the skin. Immediate pigment
darkening, which fades within minutes after exposure, is brought on by UVA and visible light. It is
caused by photooxidation of preexisting melanin and a redistribution of melanosomes from a
perinuclear position into dendrites. Epidermal thickening is mainly a UVB-induced phenomenon.
Mast cell degranulation, with release of histamine and other mast cell products, also occurs as a
result of UV radiation


97) The portion of the electromagnetic spectrum that produces a particular biologic effect is called
the:


A. Spectral Activity

B. Cutaneous Effect Spectrum

C. Action Spectrum Correct Choice

D. Effective Spectrum

E. Absorption Spectrum
The action spectrum is the wavelengths that produce a certain biologic effect. The action spectrum
for photosensitivity from exogenous chemicals is usually in the UVA range. The radiation that is
absorbed by those chemicals is called their absorption spectrum


98) Which of the following genodermatoses is NOT worsened by sunlight?

A. Darier's disease

B. Kindler syndrome

C. Rothmund-Thompson syndrome




                                                    25
D. Hartnup disease

E. Job syndromeCorrect Choice
Many genodermatoses can be exacerbated by sunlight including Darier's, Kindler, Rothmund
Thompson, Hartnup (pellagra changes


99) Actinic prurigo (AP) differs from PMLE in all of the following except:

A. Cheilitis is rare in AP and common in PMLE Correct Choice

B. The lesions of AP occur on all sun-exposed areas

C. Outbreaks of AP are not as clearly related to sun exposure

D. Lesions of AP may occur on non-sun exposed areas

E. The lesions of AP begin in childhood
Cheilitis is common in actinic prurigo and not a usual accompaniment in PMLE.


100) The MPD of Oxsoralen plus UVA is:

A. Tested on the patient’s calf or abdomen

B. Equal to one-half the patients MEDA

C. None of these answers are correct

D. Helpful in starting PUVA therapy Correct Choice

E. Measured at 24 hours
MPD stands for the minimal phototoxic dose. For Oxsoralen plus UVA, the MPD is measured at
48-72 hours. Testing is done on the upper buttock or forearm


101) The most common presentation of a patient with medication photosensitivity is:

A. Lichenoid eruptions

B. Photoonycholysis

C. Fixed erythematous patch

D. Pseudoporphyria

E. Diffuse erythema in sun-exposed areasCorrect Choice
Most patients with medication photosensitivity present with diffuse erythema in sun-exposed areas.
In some patients, the eruption is eczematous and covered areas are spared. Photoonycholysis,
lichenoid eruptions and pseudoporphyria do occur with mediation photosensitivity, but are not the
most common presentation. Fixed erythematous patch is not seen with this type of reaction


102) Which of the following is the most common photodermatosis?

A. Polymorphous light eruptionCorrect Choice

B. Hydroa vacciniforme

C. Solar urticaria

D. Actinic prurigo

E. Chronic actinic dermatitis




                                                  26
Polymorphous light eruption is the most common photodermatosis. It is a idiopathic disease that
usually appears in the first three decades of life and is more common in fair-skinned females. The
pathogenesis is unclear, but is believed to be related to a type IV hypersensitivity reaction. Most
lesions are erythematous pruritic papules, with the plaque form being less common. Lesions appear
symetrically on exposed areas after a delay of several hours to several days. Patients with mild
disease are treated with sun avoidance and a broad spectrum sunscreen. In more severe cases,
hardening and desensitization can be accomplished or antimalarials can be used for resistant cases.
For brief, sunny vacations, a short course of prednisone can be helpful.

The other options are less common forms of idiopathic photosensitivity disorders


103) UVB is about how many times more erythermogenic than UVA ?

A. 1000 Correct Choice

B. 5

C. 10000

D. 10

E. 100
Though UVB is 1000 times more erythermogenic than UVA, UVA is much more plentiful in sunlight.
Therefore, UVA does contribute to sunlight erythema


104) Phototoxicity and photoallergy from exogenous agents typically involve absorption of:

A. UVB, UVA and visible light

B. UVA Correct Choice

C. UVB

D. UVA and visible light

E. UVB and UVA
Topical and systemic agents that produce phototoxicity and/or photoallergy usually have action
spectra in the UVA range


105) The UVC portion of the electromagnetic spectrum extends from:

A. 10-200 nm

B. 200-290 nmCorrect Choice

C. 400-760 nm

D. None of these answers are correct

E. 290-320 nm
Ultraviolet C does not reach the earth's surface. It is filtered out by the ozone layer. It extends from
200-290 nm


106) Ultraviolet radition has been shown to do all of the following in in vitro and in vivo studies
EXCEPT:


A. Increase circulating levels of IL-6

B. Suppress the induction of delayed-type hypersensitivity




                                                  27
C. Decrease circulating levels of IL-1Correct Choice

D. Induce suppressor T-cells

E. Alter the ability of antigen-presenting cells to present antigen
UV-irradiated mice have been shown to have defective antigen presentation and a decreased
number of antigen-presenting cells, which prevents a normal delayed-type hypersensitivity
response. UVR causes the release of immunosuppressive factors, with induction of suppressor T-
cells and increases in circulating levels of cytokines, including IL-1, IL-6, and TNF


107) Narrowband UVB bulbs emit predominantly at:

A. 290-320 nm

B. 305 nm

C. 360 nm

D. 311 nm Correct Choice

E. 352 nm
Narrowband UVB (311-312 nm) is more effective than broadband UVB for psoriasis, vitiligo, and
other skin disorders


108) Which of the following is not true about UVB radiation?

A. Responsible for sunburn

B. Decreased with high wind velocityCorrect Choice

C. Virtually all blocked by car window glass

D. Peaks at noon

E. More intense in the summer than winter months
The UVB band extends from 290 to 320 nm. The UVB spectrum is recognized as the primary cause
of sunburn, skin cancer, and other harmful effects on human
skin. The UVA band extends from 320 to 400 nm. This spectrum is further subdivided into UVA-2
(320 to 340 nm) and UVA-1 (340 to 400 nm). The UVA spectrum is recognized as a cause of
immediate and delayed tanning reaction of skin, and several other effects including photoaging, skin
photosensitization, and immunosuppression. UVC radiation comprises wavelengths shorter than 290
nm (from 200 to 290 nm). Notably, window glass filters out ultraviolet wavelengths shorter than
320 nm, so both UVB and UVC are effectively filtered by car window glass. UVB radiation is more
intense during summer months compared to winter months and peaks during midday hours. It has
been postulated that physical factors such as high temperature, high humidity, and wind can all
increase susceptibility to UV-induced carcinogenesis


109) Which of the following statements about UVR in vivo is correct?

A. decreases circulating levels of cytokins (IL-1, IL-6 and TNF)

B. normalizes Langerhans cell morphology and function

C. normalizes cell trafficking

D. normalizes proportions of lymphocyte subtypes in peripheral blood

E. induces skin cancersCorrect Choice
UVR can induce skin cancer. UVR in vivo INCREASES circulating levels of cytokins (IL-1, IL-6 and
TNF) and alters Langerhans cell morphology, cell trafficking, and the proportion of lymphocyte
subtypes in peripheral blood



                                                   28
110) Which of the following is NOT true regarding polymorphous light eruption?

A. It may occur through windowglass, which filters out UVB

B. Usually appears in the first three decades

C. Not all exposed areas show lesions

D. May be a manifestation of a type IV hypersensitivity reaction

E. Vesicles and an eczematous dermatitis are a common presentationCorrect Choice
Polymorphous light eruption is the most common photodermatosis. It is an idiopathic disease that
usually appears in the first three decades. Pathogenesis is unclear but it may be related to a type IV
hypersensitivity reaction. Most lesions are erythematous, pruritic papules. The plaque form is less
common, and vesicles and an eczematous dermatitis are rare. Not all exposed areas show lesions,
but the same areas are affected year after year. It may improve as the summer progresses. It may
occur through windowglass, which filters out UVB


111) What is the wavelength of a Wood's light?

A. 365nmCorrect Choice

B. 410nm

C. 290nm

D. 311nm

E. 330nm
A Wood's light emits ultaviolet light at a wavelenth of 365nm and is produce by bassing light
through a Wood's filter which is composed of nickel oxide containing glass


112) The most common cause(s) of topical phototoxicity today in the United States is(are):

A. 6-methyl-coumarin

B. Musk ambrette

C. PABA and non-PABA sunscreen ingredients

D. PsoralensCorrect Choice

E. Halogenated salicylanilides
Topical phototoxicity is most commonly caused by psoralens. Topical 8-methoxypsoralen is used
therapeutically to treated psoriasis, localized vitiligo, and hand/foot eczema. Psoralens in certain
plants, fruits, and vegetables can produce phytophotodermatitis. Topical photoallergy has in the
past been caused by halogenated salicylanilides, as well as musk ambrette and 6-methyl-coumarin
in fragrances. These compounds produced photoallergic contact dermatitis, and have been removed
from marketed products. PABA, its esters, and non-PABA sunscreen ingredients are the most
common causes of topical photoallergy, not topical phototoxicity


113) Which of the following statements is true regarding ultraviolet carcinogenesis?

A. UVA is most effective in producing pyrimidine dimers, which may activate oncogenes

B. Cells from patients with actinic keratoses have normal DNA repair capacity

C. Suppressor T-cells arise in UV-irradiated hosts only after tumors have developed




                                                 29
D. UVA, when added to UVB, may accelerate carcinogenesisCorrect Choice

E. Mid-range ultraviolet radiation is less efficient in inducing neoplasia in mice that is long wave UVR
Mid-range UVR (280-320 nm) is more efficient in inducing neoplasia in mice, but long wave UVA,
when added to UVB, may accelerate carcinogenesis. Suppressor T-cells induce susceptibility to
tumors, and appear to arise in UV-irradiated hosts prior to tumors developing, thus playing a role in
carcinogenesis. Cells from patients with AKs have less DNA repair capacity than controls. UVB is
most effective in producing pyrimidine dimers, which may activate oncogenes, particularly in the
formation of BCCs and SCCs


114) A patient presents with purple polygonal pruritic papules on sun exposed areas. Which of the
following of his medication would not be suspect for causing this eruption?


A. Hydrochlorothiazide

B. Enalapril

C. Quinidine

D. FurosemideCorrect Choice

E. Fenofibrate
Furosemide (Lasix) is not a cause of lichenoid drug reactions, but can cause pseudoporphyria. The
others listed are causes of lichenoid drug reactions. Others are antimalarials,
demethylchlortetracycline and quinine


115) Which of the following drugs is commonly known to produce photosensitivity?

A. Griseofulvin

B. All of these answers are correctCorrect Choice

C. Sulfonylureas

D. Quinidine

E. None of these answers are correct
Quinidine, sulfonlyureas, and griseofulvin are all known to cause photosensitivity


116) The differential diagnosis of this patient would include:

A. Chronic actinic dermatitis

B. All of these answers are correct Correct Choice

C. Airborne contact dermatitis

D. Photoallergic contact dermatitis

E. Drug photosensitivity
NEEDS EXPLANATION


117) The action spectrum for photoallergy is mostly in which spectrum?

A. 400-760nm

B. 311-312nm

C. 200-290nm




                                                    30
D. 290-320nm

E. 320-425nmCorrect Choice
The action spectrum for photoallergic dermatoses refers to the specific wavelengths of light that
evoke the photosensitive reaction. This falls mostly within the UVA region and may spill into the
visible light region for photoallergy (320-425nm). 200-290nm refers to the UVC region; 290-320nm
refers to the UVB region; 311-312nm refers to narrowband UVB region; and 400-769nm refers to
the visible light region


118) This disease can be brought on by:

A. UVA I

B. All of these answers are correctCorrect Choice

C. Visible light

D. UVA II

E. UVB
Solar urticaria can be brought on by UVB or UVA or visible light or combinations of those
wavelengths


119) All of the following are true regarding polymorphous light eruption except:

A. Anti-Ro antibody positiveCorrect Choice

B. Hardening occurs with subsequent episodes

C. Abnormal metabolism of arachidonic acid

D. Pruritic

E. Lesions heal without scarring
Polymorphous light eruption is the most common photodermatosis that is characterized clinically by
the abnormal occurrence of pruritic, erythematous, edematous papules following exposure to UV
radiation. Lesions heal without scarring. It tends to affect women 2-3x more than men. Positive
Anti-ro antibodies should raise the suspicious for subacute cutaneous lupus erythematosus (tends
also to be less pruritic


120) Most fluorescent UV sources are:

A. Low pressure xenon arc lamps

B. Low pressure mercury vapor lamps Correct Choice

C. Low pressure argon lamps

D. High pressure tungsten lamps

E. High pressure xenon arc lamps
The mercury vapor in the fluorescent bulbs is excited by electric current. Then the mercury emits
radiation at 254 nm. This radiation is absorbed by the phosphor lining the bulb


121) Actinic prurigo (AP) differs from polymorphous light eruption (PMLE) in that:

A. Lesions of PMLE occur on all sun-exposed areas

B. Lesions of AP usually begin after puberty




                                                    31
C. Lesions of AP may persist for months, even into the winterCorrect Choice

D. Chelitis is more frequently seen in PMLE

E. Lesions of PMLE may occur on non-sun-exposed areas
Actinic prurigo (AP) may be a distinct entity, or an HLA-restricted subset of polymorphous light
eruption (PMLE). AP differs from PMLE in that the lesions of AP always begin in childhood and often
remit in puberty, the lesions of AP occur on all sun-exposed areas and may persist for months, even
into the winter, and the lesions of AP may occur on non-sun-exposed areas. In addition, outbreaks
of AP are not as clearly related to sun exposure, and chelitis is frequently seen in AP, not PMLE


122) This patient presented with hyperpigmented streaks after a vacation in the Caribbean. The
most likely diagnosis is:


A. Actinic prurigo

B. CAD

C. PMLE

D. Phytophotodermatitis Correct Choice

E. Melasma
Phytophotodermatitis requires exposure to certain plants or fruits followed by sunlight. Parsnips,
parsley, figs, limes, celery, bergamot oranges, and others contain psoralens that react with UVA.
Initially there is erythema and blistering followed by streaked hyperpigmentation


123) Which of the following is the most likely cause of photosensitivity?

A. Penicillin V

B. Quinolones

C. Sulfonamides

D. DoxycyclineCorrect Choice

E. Minocycline
Doxycycline is the tetracycline derivative most likely to cause photosensitivity. Quinolones and
sulfonamides will also cause this with ingestion. Minocycline is the least photosensitizing of the
tetracycline derivatives. Penicillin is not a common cause of photosensitivity


124) Which of the following is true regarding immediate pigment darkening?

A. Becomes prominent 48 hr after exposure

B. Contributes to constitutive skin color

C. Caused by UVA radiationCorrect Choice

D. Requires the synthesis of new melanin

E. Prominent in lightly pigmented individuals
Tanning develops in two phases, early (transitory) and late (stable). The immediate darkening is in
response to UVA and is related to photo-oxidation of pre-existing melanin


125) Which of the following statements is true regarding UV radiation, erythema, and
pigmentation?




                                                  32
A. Delayed tanning, which becomes visible about 72 hours after exposure, is largely brought on by
UVA.

B. UVB erythema reaches a maximum in 24-36 hours

C. The chromophores involved with UVB erythema are melanosomes

D. Immediate pigment darkening is brought on by UVA and visible lightCorrect Choice

E. Immediate pigment darkening fades within 12-24 hours after exposure
UVB in natural sunlight is the main contributor to erythema. UVB erythema reaches a maximum in
6-24 hours. The chromophores involved with UVB erythema are not clear but appear to involve
nucleic acids. Immediate pigment darkening is brought on by UVA and visible light, and fades within
minutes after exposure. Delayed tanning becomes visible about 72 hours after UVB exposure. UVA
contributes to a lesser extent to delayed tanning


126) The main condition on the differential for polymorphous light eruption is lupus erythematosus.
Which of the following tests should NOT be performed to help make this distinction?


A. Anti-SSA

B. Antinuclear antibody

C. SED rateCorrect Choice

D. Anti-SSB

E. Skin biopsy for routine staining and direct immunofluorescence
All of the listed tests are helpful in distinguishing between PMLE and lupus except a SED rate, which
is a non-specific marker of systemic inflammation


127) As a result of ultraviolet radiation:

A. Prostaglandins are increased

B. None of these answers are correct

C. There is mast cell degranulation and release of histamine

D. Epidermal thickening occurs

E. All of these answers are correctCorrect Choice
Effects of ultraviolet radiation include mast cell degranulation with release of histamine and other
mast cell products, increases in certain prostaglandins and interleukins, and epidermal thickening,
which is mainly a UVB-induced phenomenon


128) Which spectrum of UV is responsible for the conversion of 7-dehydrocholesterol in the skin to
pre-vitamin D3?


A. 290-320 nmCorrect Choice

B. 10-200 nm

C. 340-400 nm

D. 320-340 nm

E. 200-290 nm
Ultraviolet B with the spectrum of 290-320 nm is responsible for the conversion of 7-
dehydrocholesterol in the skin to pre-vitamin D3




                                                    33
129) What range of ultraviolet radiation has been shown to be most efficient in inducing neoplasia
in mice?


A. 280-320nmCorrect Choice

B. >760nm

C. 400-760nm

D. 340-400nm

E. 320-340nm
Midrange UVR 280-320nm (UVB range) has been shown to be most efficient in inducing neoplasia in
mice. Long-wave UVA, when added to UVB may accelerate carcinogenesis


130) A normal MEDB on untanned Caucasian skin ranges from approximately:

A. 20-70 mJ/cm2 Correct Choice

B. 2-6 mJ/cm2

C. 140-200 mJ/cm2

D. 15-40 mJ/cm2

E. 70-140 mJ/cm2
The MEDB can vary from institution to institution. In one institution, it may range from 20-70
mJ/cm2. In another institution, it may range from 30-90 mJ/cm2


131) Phototoxicity from exogenous agents is characterized by all of the following except:

A. There may be apoptotic cells histologically

B. There is cross-reactivity to similar exogenous agents Correct Choice

C. It appears as an exaggerated sunburn and heals with hyperpigmentation

D. Occurs in most individuals given enough of the drug and enough UVR

E. It can occur on first exposure to drug and UVR
With photoallergy, not phototoxicity, there can be cross-reactivity among similar agents


132) A patient with a personal and family history of multiple fibrofolliculomas may have:

A. Tuberous sclerosis

B. Muir-Torre syndrome

C. Gardner's syndrome

D. Birt-Hogg-Dube syndromeCorrect Choice

E. Basal cell nevus syndrome
Fibrofolliculomas are small, benign, yellow or flesh colored papules that are usually inherited in an
autosomal dominant fashion and have a predilection for the face, neck and upper trunk. Birt-Hogg-
Dube syndrome is an autosomal dominant condition characterized by multiple fibrofolliculomas,
trichodiscomas, acrochordons, collagenomas. This condition is important to recognize due to its
association with renal cell carcinoma. In addition, lung cysts and bullous emphysema are also
features of the condition with spontaneous pneumothorax being a potential complication.
Muir-Torre syndrome is an autosomal dominant condition caused by a defect in hMSH2 gene. This
condition is associated with sebaceous neoplasms including sebaceous carcinoma, sebaceous



                                                    34
hyperplasia, sebaceous epithelioma, sebaceous adenoma as well as keratoacanthomas. These can
be markers for underlying malignancy in this condition; notably adenocarcinoma of the colon,
breast, urinary tract, lung and endometrium.
Gardner syndrome is an autosomal dominant condition caused by a defect in the APC familial
adenomatous polyposis gene in which patients have multiple hamartomatous polyps of the colon
with a high rate of malignant transformation. Cutaneous clues to the diagnosis include multiple
epidermoid cysts, fibromas, and desmoid tumors. Other manifestations include osteomas,
supernumary teeth, and congential hypertrophy of the retinal pigment epithelium.
Tuberous sclerosis, (TS),also known as Bourneville's syndrome is caused by defects in hamartin and
tuberin, found on chromosome 9 and 16, respectively. Patients with TS are at increased risk of
muliple neoplasms including retinal hamartomas, angiomyolipomas, and cardiac rhabdomyomas.
Cutaneous manifestations include ash-leaf macules, shagreen patchs, café-au-lait macules, confetti
macules, facial angiofibromas, and periungual fibromas.
As the name implies, basal cell nevus syndrome is associated with multiple basal cell carcinomas. In
addition palmoplantar pitting, multiple milia and epidermoid cysts are seen. It is autosomal
dominantly inherited and caused by a defect in the patched gene




                                                35

Contenu connexe

Tendances

ETAS_MCQ_11 disorder of hair and nails
ETAS_MCQ_11 disorder of hair and nailsETAS_MCQ_11 disorder of hair and nails
ETAS_MCQ_11 disorder of hair and nailsDerma202
 
ETAS_MCQ_09 pediatric dermatology
ETAS_MCQ_09 pediatric dermatologyETAS_MCQ_09 pediatric dermatology
ETAS_MCQ_09 pediatric dermatologyDerma202
 
ETAS_MCQ_03 b genodermatoses
ETAS_MCQ_03 b genodermatosesETAS_MCQ_03 b genodermatoses
ETAS_MCQ_03 b genodermatosesDerma202
 
ETAS_MCQ_03 a genodermatoses
ETAS_MCQ_03 a genodermatosesETAS_MCQ_03 a genodermatoses
ETAS_MCQ_03 a genodermatosesDerma202
 
Mock question paper for neet pg, usmle, plab and fmge (mci screening exam) on...
Mock question paper for neet pg, usmle, plab and fmge (mci screening exam) on...Mock question paper for neet pg, usmle, plab and fmge (mci screening exam) on...
Mock question paper for neet pg, usmle, plab and fmge (mci screening exam) on...Medico Apps
 
100 mc qsintropicalmedicine
100 mc qsintropicalmedicine100 mc qsintropicalmedicine
100 mc qsintropicalmedicineElyas Mohammed
 
Previous year question on lichen planus based on neet pg, usmle, plab and fmg...
Previous year question on lichen planus based on neet pg, usmle, plab and fmg...Previous year question on lichen planus based on neet pg, usmle, plab and fmg...
Previous year question on lichen planus based on neet pg, usmle, plab and fmg...Abhishek Gupta
 
Previous year question on leptospirosis based on neet pg, usmle, plab and fmg...
Previous year question on leptospirosis based on neet pg, usmle, plab and fmg...Previous year question on leptospirosis based on neet pg, usmle, plab and fmg...
Previous year question on leptospirosis based on neet pg, usmle, plab and fmg...Medico Apps
 
50 1339
50 133950 1339
50 1339edcito
 
Ocular inflammatory disease and ocular tuberculosis in a cohort of patients c...
Ocular inflammatory disease and ocular tuberculosis in a cohort of patients c...Ocular inflammatory disease and ocular tuberculosis in a cohort of patients c...
Ocular inflammatory disease and ocular tuberculosis in a cohort of patients c...Dr.Samsuddin Khan
 
Microbiology MCQs
Microbiology MCQsMicrobiology MCQs
Microbiology MCQsPankaj Jha
 
Parasitology revision 2016 mod ug
Parasitology revision 2016 mod ugParasitology revision 2016 mod ug
Parasitology revision 2016 mod ugMohammed Sarhan
 
Tree-man disease (Epidermodysplasia Verruciformis)
Tree-man disease (Epidermodysplasia Verruciformis)Tree-man disease (Epidermodysplasia Verruciformis)
Tree-man disease (Epidermodysplasia Verruciformis)Ashish Kumar
 
Parasitology questions and answers
Parasitology questions and answersParasitology questions and answers
Parasitology questions and answersNayetuni Eddy
 
Vesiculo bullous II
Vesiculo bullous IIVesiculo bullous II
Vesiculo bullous IIIAU Dent
 
New approaches for medical management of Chronic Granulomatous Disease.
New approaches for medical management of Chronic Granulomatous Disease.New approaches for medical management of Chronic Granulomatous Disease.
New approaches for medical management of Chronic Granulomatous Disease.Dmitri Popov
 
Pv0811 werner ce_blastomycosis-10-8-2011
Pv0811 werner ce_blastomycosis-10-8-2011Pv0811 werner ce_blastomycosis-10-8-2011
Pv0811 werner ce_blastomycosis-10-8-2011Minakata Jin
 
Vohwinkel Syndrome vs. KID Syndrome
Vohwinkel Syndrome vs. KID Syndrome Vohwinkel Syndrome vs. KID Syndrome
Vohwinkel Syndrome vs. KID Syndrome JenniferArmstrong6
 
Tubercular uveitis
Tubercular uveitisTubercular uveitis
Tubercular uveitisBipin Bista
 

Tendances (20)

ETAS_MCQ_11 disorder of hair and nails
ETAS_MCQ_11 disorder of hair and nailsETAS_MCQ_11 disorder of hair and nails
ETAS_MCQ_11 disorder of hair and nails
 
ETAS_MCQ_09 pediatric dermatology
ETAS_MCQ_09 pediatric dermatologyETAS_MCQ_09 pediatric dermatology
ETAS_MCQ_09 pediatric dermatology
 
ETAS_MCQ_03 b genodermatoses
ETAS_MCQ_03 b genodermatosesETAS_MCQ_03 b genodermatoses
ETAS_MCQ_03 b genodermatoses
 
ETAS_MCQ_03 a genodermatoses
ETAS_MCQ_03 a genodermatosesETAS_MCQ_03 a genodermatoses
ETAS_MCQ_03 a genodermatoses
 
Mock question paper for neet pg, usmle, plab and fmge (mci screening exam) on...
Mock question paper for neet pg, usmle, plab and fmge (mci screening exam) on...Mock question paper for neet pg, usmle, plab and fmge (mci screening exam) on...
Mock question paper for neet pg, usmle, plab and fmge (mci screening exam) on...
 
100 mc qsintropicalmedicine
100 mc qsintropicalmedicine100 mc qsintropicalmedicine
100 mc qsintropicalmedicine
 
Previous year question on lichen planus based on neet pg, usmle, plab and fmg...
Previous year question on lichen planus based on neet pg, usmle, plab and fmg...Previous year question on lichen planus based on neet pg, usmle, plab and fmg...
Previous year question on lichen planus based on neet pg, usmle, plab and fmg...
 
Previous year question on leptospirosis based on neet pg, usmle, plab and fmg...
Previous year question on leptospirosis based on neet pg, usmle, plab and fmg...Previous year question on leptospirosis based on neet pg, usmle, plab and fmg...
Previous year question on leptospirosis based on neet pg, usmle, plab and fmg...
 
50 1339
50 133950 1339
50 1339
 
Ocular inflammatory disease and ocular tuberculosis in a cohort of patients c...
Ocular inflammatory disease and ocular tuberculosis in a cohort of patients c...Ocular inflammatory disease and ocular tuberculosis in a cohort of patients c...
Ocular inflammatory disease and ocular tuberculosis in a cohort of patients c...
 
Open Journal of Pediatrics & Neonatal Care
Open Journal of Pediatrics & Neonatal CareOpen Journal of Pediatrics & Neonatal Care
Open Journal of Pediatrics & Neonatal Care
 
Microbiology MCQs
Microbiology MCQsMicrobiology MCQs
Microbiology MCQs
 
Parasitology revision 2016 mod ug
Parasitology revision 2016 mod ugParasitology revision 2016 mod ug
Parasitology revision 2016 mod ug
 
Tree-man disease (Epidermodysplasia Verruciformis)
Tree-man disease (Epidermodysplasia Verruciformis)Tree-man disease (Epidermodysplasia Verruciformis)
Tree-man disease (Epidermodysplasia Verruciformis)
 
Parasitology questions and answers
Parasitology questions and answersParasitology questions and answers
Parasitology questions and answers
 
Vesiculo bullous II
Vesiculo bullous IIVesiculo bullous II
Vesiculo bullous II
 
New approaches for medical management of Chronic Granulomatous Disease.
New approaches for medical management of Chronic Granulomatous Disease.New approaches for medical management of Chronic Granulomatous Disease.
New approaches for medical management of Chronic Granulomatous Disease.
 
Pv0811 werner ce_blastomycosis-10-8-2011
Pv0811 werner ce_blastomycosis-10-8-2011Pv0811 werner ce_blastomycosis-10-8-2011
Pv0811 werner ce_blastomycosis-10-8-2011
 
Vohwinkel Syndrome vs. KID Syndrome
Vohwinkel Syndrome vs. KID Syndrome Vohwinkel Syndrome vs. KID Syndrome
Vohwinkel Syndrome vs. KID Syndrome
 
Tubercular uveitis
Tubercular uveitisTubercular uveitis
Tubercular uveitis
 

Similaire à ETAS_MCQ_13 photobiology and photosensitivity disorders

Dermatology MCQ and AAFP.pptx
Dermatology MCQ and AAFP.pptxDermatology MCQ and AAFP.pptx
Dermatology MCQ and AAFP.pptxAbdulaziz Bagasi
 
FSL for atopic dermatitis
FSL for atopic dermatitisFSL for atopic dermatitis
FSL for atopic dermatitisHans Kim
 
Modalities Poster Presentation
Modalities Poster PresentationModalities Poster Presentation
Modalities Poster PresentationLauren Slayton
 
Ultraviolet principals and applications
Ultraviolet  principals and applicationsUltraviolet  principals and applications
Ultraviolet principals and applicationsmahmood wajeeh
 
Internal Medicine Image Challenge MCQs
Internal Medicine Image Challenge MCQsInternal Medicine Image Challenge MCQs
Internal Medicine Image Challenge MCQsSherif Elbadrawy
 
233240482 antiradiation-uv-vaccine
233240482 antiradiation-uv-vaccine233240482 antiradiation-uv-vaccine
233240482 antiradiation-uv-vaccineDmitri Popov
 
AHS13 Anastasia Boulais — Is Sun Worshipping Increasing Your Risk of Melanoma?
AHS13 Anastasia Boulais — Is Sun Worshipping Increasing Your Risk of Melanoma? AHS13 Anastasia Boulais — Is Sun Worshipping Increasing Your Risk of Melanoma?
AHS13 Anastasia Boulais — Is Sun Worshipping Increasing Your Risk of Melanoma? Ancestral Health Society
 
Acute Radiation Syndrome
Acute Radiation SyndromeAcute Radiation Syndrome
Acute Radiation SyndromeSun Yai-Cheng
 
Chronic actinic dermatitis
Chronic actinic dermatitisChronic actinic dermatitis
Chronic actinic dermatitisChandan N
 
ROLE OF RADIATIONS IN MEDICAL TREATMENT
ROLE OF RADIATIONS IN MEDICAL TREATMENTROLE OF RADIATIONS IN MEDICAL TREATMENT
ROLE OF RADIATIONS IN MEDICAL TREATMENTshahidfarid10
 
Self-assessment in optic and refraction by prof Chua, dr. Chieng, dr.ngo and ...
Self-assessment in optic and refraction by prof Chua, dr. Chieng, dr.ngo and ...Self-assessment in optic and refraction by prof Chua, dr. Chieng, dr.ngo and ...
Self-assessment in optic and refraction by prof Chua, dr. Chieng, dr.ngo and ...Mero Eye
 
Phototherapy treatment protocol
Phototherapy treatment protocolPhototherapy treatment protocol
Phototherapy treatment protocolDerma202
 
Internal Medicine Board Review
Internal Medicine  Board ReviewInternal Medicine  Board Review
Internal Medicine Board Reviewjcm MD
 
Dr. Treacy's Dermatology Casebook No 1
Dr. Treacy's Dermatology Casebook No 1 Dr. Treacy's Dermatology Casebook No 1
Dr. Treacy's Dermatology Casebook No 1 Dr. Patrick J. Treacy
 

Similaire à ETAS_MCQ_13 photobiology and photosensitivity disorders (20)

Dermatology MCQ and AAFP.pptx
Dermatology MCQ and AAFP.pptxDermatology MCQ and AAFP.pptx
Dermatology MCQ and AAFP.pptx
 
FSL for atopic dermatitis
FSL for atopic dermatitisFSL for atopic dermatitis
FSL for atopic dermatitis
 
Ganyang MCQ Dermatology
Ganyang MCQ DermatologyGanyang MCQ Dermatology
Ganyang MCQ Dermatology
 
Modalities Poster Presentation
Modalities Poster PresentationModalities Poster Presentation
Modalities Poster Presentation
 
Drill 3
Drill 3Drill 3
Drill 3
 
Ultraviolet principals and applications
Ultraviolet  principals and applicationsUltraviolet  principals and applications
Ultraviolet principals and applications
 
Internal Medicine Image Challenge MCQs
Internal Medicine Image Challenge MCQsInternal Medicine Image Challenge MCQs
Internal Medicine Image Challenge MCQs
 
233240482 antiradiation-uv-vaccine
233240482 antiradiation-uv-vaccine233240482 antiradiation-uv-vaccine
233240482 antiradiation-uv-vaccine
 
AHS13 Anastasia Boulais — Is Sun Worshipping Increasing Your Risk of Melanoma?
AHS13 Anastasia Boulais — Is Sun Worshipping Increasing Your Risk of Melanoma? AHS13 Anastasia Boulais — Is Sun Worshipping Increasing Your Risk of Melanoma?
AHS13 Anastasia Boulais — Is Sun Worshipping Increasing Your Risk of Melanoma?
 
Acute Radiation Syndrome
Acute Radiation SyndromeAcute Radiation Syndrome
Acute Radiation Syndrome
 
Chronic actinic dermatitis
Chronic actinic dermatitisChronic actinic dermatitis
Chronic actinic dermatitis
 
MRCP MOCK EXAM
MRCP MOCK EXAMMRCP MOCK EXAM
MRCP MOCK EXAM
 
EED 1.pptx
EED 1.pptxEED 1.pptx
EED 1.pptx
 
leprosy-150919182725-lva1-app6892.pdf
leprosy-150919182725-lva1-app6892.pdfleprosy-150919182725-lva1-app6892.pdf
leprosy-150919182725-lva1-app6892.pdf
 
Leprosy
LeprosyLeprosy
Leprosy
 
ROLE OF RADIATIONS IN MEDICAL TREATMENT
ROLE OF RADIATIONS IN MEDICAL TREATMENTROLE OF RADIATIONS IN MEDICAL TREATMENT
ROLE OF RADIATIONS IN MEDICAL TREATMENT
 
Self-assessment in optic and refraction by prof Chua, dr. Chieng, dr.ngo and ...
Self-assessment in optic and refraction by prof Chua, dr. Chieng, dr.ngo and ...Self-assessment in optic and refraction by prof Chua, dr. Chieng, dr.ngo and ...
Self-assessment in optic and refraction by prof Chua, dr. Chieng, dr.ngo and ...
 
Phototherapy treatment protocol
Phototherapy treatment protocolPhototherapy treatment protocol
Phototherapy treatment protocol
 
Internal Medicine Board Review
Internal Medicine  Board ReviewInternal Medicine  Board Review
Internal Medicine Board Review
 
Dr. Treacy's Dermatology Casebook No 1
Dr. Treacy's Dermatology Casebook No 1 Dr. Treacy's Dermatology Casebook No 1
Dr. Treacy's Dermatology Casebook No 1
 

Plus de Derma202

Histopathplogical photos
Histopathplogical photosHistopathplogical photos
Histopathplogical photosDerma202
 
Slide study from ETAS
Slide  study from ETASSlide  study from ETAS
Slide study from ETASDerma202
 
Arab board primary exam in dermatology 2012
Arab board primary exam  in dermatology 2012Arab board primary exam  in dermatology 2012
Arab board primary exam in dermatology 2012Derma202
 
Dermatology
DermatologyDermatology
DermatologyDerma202
 
ETAS_MCQ_02 immunodermatology
ETAS_MCQ_02 immunodermatologyETAS_MCQ_02 immunodermatology
ETAS_MCQ_02 immunodermatologyDerma202
 
ETAS_MCQ_01 structures of skin
ETAS_MCQ_01 structures of skinETAS_MCQ_01 structures of skin
ETAS_MCQ_01 structures of skinDerma202
 
Derm handbook for medical students and junior doctors 2010
Derm handbook for medical students and junior doctors 2010Derm handbook for medical students and junior doctors 2010
Derm handbook for medical students and junior doctors 2010Derma202
 

Plus de Derma202 (7)

Histopathplogical photos
Histopathplogical photosHistopathplogical photos
Histopathplogical photos
 
Slide study from ETAS
Slide  study from ETASSlide  study from ETAS
Slide study from ETAS
 
Arab board primary exam in dermatology 2012
Arab board primary exam  in dermatology 2012Arab board primary exam  in dermatology 2012
Arab board primary exam in dermatology 2012
 
Dermatology
DermatologyDermatology
Dermatology
 
ETAS_MCQ_02 immunodermatology
ETAS_MCQ_02 immunodermatologyETAS_MCQ_02 immunodermatology
ETAS_MCQ_02 immunodermatology
 
ETAS_MCQ_01 structures of skin
ETAS_MCQ_01 structures of skinETAS_MCQ_01 structures of skin
ETAS_MCQ_01 structures of skin
 
Derm handbook for medical students and junior doctors 2010
Derm handbook for medical students and junior doctors 2010Derm handbook for medical students and junior doctors 2010
Derm handbook for medical students and junior doctors 2010
 

Dernier

College Call Girls Vyasarpadi Whatsapp 7001305949 Independent Escort Service
College Call Girls Vyasarpadi Whatsapp 7001305949 Independent Escort ServiceCollege Call Girls Vyasarpadi Whatsapp 7001305949 Independent Escort Service
College Call Girls Vyasarpadi Whatsapp 7001305949 Independent Escort ServiceNehru place Escorts
 
Call Girls Service in Bommanahalli - 7001305949 with real photos and phone nu...
Call Girls Service in Bommanahalli - 7001305949 with real photos and phone nu...Call Girls Service in Bommanahalli - 7001305949 with real photos and phone nu...
Call Girls Service in Bommanahalli - 7001305949 with real photos and phone nu...narwatsonia7
 
Call Girl Lucknow Mallika 7001305949 Independent Escort Service Lucknow
Call Girl Lucknow Mallika 7001305949 Independent Escort Service LucknowCall Girl Lucknow Mallika 7001305949 Independent Escort Service Lucknow
Call Girl Lucknow Mallika 7001305949 Independent Escort Service Lucknownarwatsonia7
 
Housewife Call Girls Hsr Layout - Call 7001305949 Rs-3500 with A/C Room Cash ...
Housewife Call Girls Hsr Layout - Call 7001305949 Rs-3500 with A/C Room Cash ...Housewife Call Girls Hsr Layout - Call 7001305949 Rs-3500 with A/C Room Cash ...
Housewife Call Girls Hsr Layout - Call 7001305949 Rs-3500 with A/C Room Cash ...narwatsonia7
 
Call Girl Bangalore Nandini 7001305949 Independent Escort Service Bangalore
Call Girl Bangalore Nandini 7001305949 Independent Escort Service BangaloreCall Girl Bangalore Nandini 7001305949 Independent Escort Service Bangalore
Call Girl Bangalore Nandini 7001305949 Independent Escort Service Bangalorenarwatsonia7
 
Call Girls Hsr Layout Just Call 7001305949 Top Class Call Girl Service Available
Call Girls Hsr Layout Just Call 7001305949 Top Class Call Girl Service AvailableCall Girls Hsr Layout Just Call 7001305949 Top Class Call Girl Service Available
Call Girls Hsr Layout Just Call 7001305949 Top Class Call Girl Service Availablenarwatsonia7
 
97111 47426 Call Girls In Delhi MUNIRKAA
97111 47426 Call Girls In Delhi MUNIRKAA97111 47426 Call Girls In Delhi MUNIRKAA
97111 47426 Call Girls In Delhi MUNIRKAAjennyeacort
 
Book Call Girls in Yelahanka - For 7001305949 Cheap & Best with original Photos
Book Call Girls in Yelahanka - For 7001305949 Cheap & Best with original PhotosBook Call Girls in Yelahanka - For 7001305949 Cheap & Best with original Photos
Book Call Girls in Yelahanka - For 7001305949 Cheap & Best with original Photosnarwatsonia7
 
Call Girl Service Bidadi - For 7001305949 Cheap & Best with original Photos
Call Girl Service Bidadi - For 7001305949 Cheap & Best with original PhotosCall Girl Service Bidadi - For 7001305949 Cheap & Best with original Photos
Call Girl Service Bidadi - For 7001305949 Cheap & Best with original Photosnarwatsonia7
 
Call Girls Kanakapura Road Just Call 7001305949 Top Class Call Girl Service A...
Call Girls Kanakapura Road Just Call 7001305949 Top Class Call Girl Service A...Call Girls Kanakapura Road Just Call 7001305949 Top Class Call Girl Service A...
Call Girls Kanakapura Road Just Call 7001305949 Top Class Call Girl Service A...narwatsonia7
 
Bangalore Call Girls Marathahalli 📞 9907093804 High Profile Service 100% Safe
Bangalore Call Girls Marathahalli 📞 9907093804 High Profile Service 100% SafeBangalore Call Girls Marathahalli 📞 9907093804 High Profile Service 100% Safe
Bangalore Call Girls Marathahalli 📞 9907093804 High Profile Service 100% Safenarwatsonia7
 
Call Girls Hebbal Just Call 7001305949 Top Class Call Girl Service Available
Call Girls Hebbal Just Call 7001305949 Top Class Call Girl Service AvailableCall Girls Hebbal Just Call 7001305949 Top Class Call Girl Service Available
Call Girls Hebbal Just Call 7001305949 Top Class Call Girl Service Availablenarwatsonia7
 
Glomerular Filtration rate and its determinants.pptx
Glomerular Filtration rate and its determinants.pptxGlomerular Filtration rate and its determinants.pptx
Glomerular Filtration rate and its determinants.pptxDr.Nusrat Tariq
 
Asthma Review - GINA guidelines summary 2024
Asthma Review - GINA guidelines summary 2024Asthma Review - GINA guidelines summary 2024
Asthma Review - GINA guidelines summary 2024Gabriel Guevara MD
 
Russian Call Girls Chickpet - 7001305949 Booking and charges genuine rate for...
Russian Call Girls Chickpet - 7001305949 Booking and charges genuine rate for...Russian Call Girls Chickpet - 7001305949 Booking and charges genuine rate for...
Russian Call Girls Chickpet - 7001305949 Booking and charges genuine rate for...narwatsonia7
 
Low Rate Call Girls Mumbai Suman 9910780858 Independent Escort Service Mumbai
Low Rate Call Girls Mumbai Suman 9910780858 Independent Escort Service MumbaiLow Rate Call Girls Mumbai Suman 9910780858 Independent Escort Service Mumbai
Low Rate Call Girls Mumbai Suman 9910780858 Independent Escort Service Mumbaisonalikaur4
 
Mumbai Call Girls Service 9910780858 Real Russian Girls Looking Models
Mumbai Call Girls Service 9910780858 Real Russian Girls Looking ModelsMumbai Call Girls Service 9910780858 Real Russian Girls Looking Models
Mumbai Call Girls Service 9910780858 Real Russian Girls Looking Modelssonalikaur4
 
VIP Call Girls Mumbai Arpita 9910780858 Independent Escort Service Mumbai
VIP Call Girls Mumbai Arpita 9910780858 Independent Escort Service MumbaiVIP Call Girls Mumbai Arpita 9910780858 Independent Escort Service Mumbai
VIP Call Girls Mumbai Arpita 9910780858 Independent Escort Service Mumbaisonalikaur4
 
College Call Girls Pune Mira 9907093804 Short 1500 Night 6000 Best call girls...
College Call Girls Pune Mira 9907093804 Short 1500 Night 6000 Best call girls...College Call Girls Pune Mira 9907093804 Short 1500 Night 6000 Best call girls...
College Call Girls Pune Mira 9907093804 Short 1500 Night 6000 Best call girls...Miss joya
 

Dernier (20)

College Call Girls Vyasarpadi Whatsapp 7001305949 Independent Escort Service
College Call Girls Vyasarpadi Whatsapp 7001305949 Independent Escort ServiceCollege Call Girls Vyasarpadi Whatsapp 7001305949 Independent Escort Service
College Call Girls Vyasarpadi Whatsapp 7001305949 Independent Escort Service
 
Call Girls Service in Bommanahalli - 7001305949 with real photos and phone nu...
Call Girls Service in Bommanahalli - 7001305949 with real photos and phone nu...Call Girls Service in Bommanahalli - 7001305949 with real photos and phone nu...
Call Girls Service in Bommanahalli - 7001305949 with real photos and phone nu...
 
sauth delhi call girls in Bhajanpura 🔝 9953056974 🔝 escort Service
sauth delhi call girls in Bhajanpura 🔝 9953056974 🔝 escort Servicesauth delhi call girls in Bhajanpura 🔝 9953056974 🔝 escort Service
sauth delhi call girls in Bhajanpura 🔝 9953056974 🔝 escort Service
 
Call Girl Lucknow Mallika 7001305949 Independent Escort Service Lucknow
Call Girl Lucknow Mallika 7001305949 Independent Escort Service LucknowCall Girl Lucknow Mallika 7001305949 Independent Escort Service Lucknow
Call Girl Lucknow Mallika 7001305949 Independent Escort Service Lucknow
 
Housewife Call Girls Hsr Layout - Call 7001305949 Rs-3500 with A/C Room Cash ...
Housewife Call Girls Hsr Layout - Call 7001305949 Rs-3500 with A/C Room Cash ...Housewife Call Girls Hsr Layout - Call 7001305949 Rs-3500 with A/C Room Cash ...
Housewife Call Girls Hsr Layout - Call 7001305949 Rs-3500 with A/C Room Cash ...
 
Call Girl Bangalore Nandini 7001305949 Independent Escort Service Bangalore
Call Girl Bangalore Nandini 7001305949 Independent Escort Service BangaloreCall Girl Bangalore Nandini 7001305949 Independent Escort Service Bangalore
Call Girl Bangalore Nandini 7001305949 Independent Escort Service Bangalore
 
Call Girls Hsr Layout Just Call 7001305949 Top Class Call Girl Service Available
Call Girls Hsr Layout Just Call 7001305949 Top Class Call Girl Service AvailableCall Girls Hsr Layout Just Call 7001305949 Top Class Call Girl Service Available
Call Girls Hsr Layout Just Call 7001305949 Top Class Call Girl Service Available
 
97111 47426 Call Girls In Delhi MUNIRKAA
97111 47426 Call Girls In Delhi MUNIRKAA97111 47426 Call Girls In Delhi MUNIRKAA
97111 47426 Call Girls In Delhi MUNIRKAA
 
Book Call Girls in Yelahanka - For 7001305949 Cheap & Best with original Photos
Book Call Girls in Yelahanka - For 7001305949 Cheap & Best with original PhotosBook Call Girls in Yelahanka - For 7001305949 Cheap & Best with original Photos
Book Call Girls in Yelahanka - For 7001305949 Cheap & Best with original Photos
 
Call Girl Service Bidadi - For 7001305949 Cheap & Best with original Photos
Call Girl Service Bidadi - For 7001305949 Cheap & Best with original PhotosCall Girl Service Bidadi - For 7001305949 Cheap & Best with original Photos
Call Girl Service Bidadi - For 7001305949 Cheap & Best with original Photos
 
Call Girls Kanakapura Road Just Call 7001305949 Top Class Call Girl Service A...
Call Girls Kanakapura Road Just Call 7001305949 Top Class Call Girl Service A...Call Girls Kanakapura Road Just Call 7001305949 Top Class Call Girl Service A...
Call Girls Kanakapura Road Just Call 7001305949 Top Class Call Girl Service A...
 
Bangalore Call Girls Marathahalli 📞 9907093804 High Profile Service 100% Safe
Bangalore Call Girls Marathahalli 📞 9907093804 High Profile Service 100% SafeBangalore Call Girls Marathahalli 📞 9907093804 High Profile Service 100% Safe
Bangalore Call Girls Marathahalli 📞 9907093804 High Profile Service 100% Safe
 
Call Girls Hebbal Just Call 7001305949 Top Class Call Girl Service Available
Call Girls Hebbal Just Call 7001305949 Top Class Call Girl Service AvailableCall Girls Hebbal Just Call 7001305949 Top Class Call Girl Service Available
Call Girls Hebbal Just Call 7001305949 Top Class Call Girl Service Available
 
Glomerular Filtration rate and its determinants.pptx
Glomerular Filtration rate and its determinants.pptxGlomerular Filtration rate and its determinants.pptx
Glomerular Filtration rate and its determinants.pptx
 
Asthma Review - GINA guidelines summary 2024
Asthma Review - GINA guidelines summary 2024Asthma Review - GINA guidelines summary 2024
Asthma Review - GINA guidelines summary 2024
 
Russian Call Girls Chickpet - 7001305949 Booking and charges genuine rate for...
Russian Call Girls Chickpet - 7001305949 Booking and charges genuine rate for...Russian Call Girls Chickpet - 7001305949 Booking and charges genuine rate for...
Russian Call Girls Chickpet - 7001305949 Booking and charges genuine rate for...
 
Low Rate Call Girls Mumbai Suman 9910780858 Independent Escort Service Mumbai
Low Rate Call Girls Mumbai Suman 9910780858 Independent Escort Service MumbaiLow Rate Call Girls Mumbai Suman 9910780858 Independent Escort Service Mumbai
Low Rate Call Girls Mumbai Suman 9910780858 Independent Escort Service Mumbai
 
Mumbai Call Girls Service 9910780858 Real Russian Girls Looking Models
Mumbai Call Girls Service 9910780858 Real Russian Girls Looking ModelsMumbai Call Girls Service 9910780858 Real Russian Girls Looking Models
Mumbai Call Girls Service 9910780858 Real Russian Girls Looking Models
 
VIP Call Girls Mumbai Arpita 9910780858 Independent Escort Service Mumbai
VIP Call Girls Mumbai Arpita 9910780858 Independent Escort Service MumbaiVIP Call Girls Mumbai Arpita 9910780858 Independent Escort Service Mumbai
VIP Call Girls Mumbai Arpita 9910780858 Independent Escort Service Mumbai
 
College Call Girls Pune Mira 9907093804 Short 1500 Night 6000 Best call girls...
College Call Girls Pune Mira 9907093804 Short 1500 Night 6000 Best call girls...College Call Girls Pune Mira 9907093804 Short 1500 Night 6000 Best call girls...
College Call Girls Pune Mira 9907093804 Short 1500 Night 6000 Best call girls...
 

ETAS_MCQ_13 photobiology and photosensitivity disorders

  • 1. Photobiology and Photosensitivity Disorders 1) What mutation is responsible for this clinical presentation in a patient with thyroid dysfunction and chronic candidal infections? A. LYST B. PTEN C. AIRECorrect Choice D. TRP1 E. fumarate hydratase APECED syndrome named for the clinical features of autoimmune polyendocrinopathy, candidiasis, ectodermal dystrophy and is caused by a mutation in AIRE (autoimmune regulator). 13% of these patient have vitigo 2) Treatment of this condition might include: A. Cyclosporin B. Antimalarials Correct Choice C. Cytoxan D. Azathioprine E. All of these answers are correct Sun avoidance, sunblocks, protective clothing, and topical steroids are sufficient for most patients with PMLE. Other patients may require hardening with UVB or PUVA. Rare patients require antimalarials 3) Immediate pigment darkening: A. Is caused by an increase in tyrosinase activity B. Is predominately brought on by UVA and visible light Correct Choice C. Is associated with an increase in melanocyte number D. Start 45-60 minutes after exposure E. Is predominately brought on by UVB Immediate pigment darkening appears almost as soon as irradiation occurs. It is due to photo- oxidation of preexisting melanin 4) A patient present with blue-gray pigmentation on sun-exposed areas but does not have involvement of the sclerae, lunulae or mucous membranes. Which of the following medications is could be causing this pigmentation? A. Sertraline B. Fluoxetine C. ChlorpromazineCorrect Choice D. Clindamycin E. Ciprofloxacin 1
  • 2. Amiodarone, chlorpromazine and tricyclic antidepressants all are capable of causing blue-gray pigmentation on sun-exposed areas without involvement of the sclerae, lunulae or mucous membranes 5) UVB acts on what compound to form pre-Vitamin D3 ? A. 7-deoxycholesterol B. 9-hydrocholesterol C. 5-dehydrocholesterol D. 7-hydrocholesterol Correct Choice E. DNA UVB converts 7-dehydrocholesterol in the skin to pre-Vitamin D3, which then thermally isomerizes to form Vitamin D3 6) Which of the following statements regarding phototesting is correct? A. To test for a visible light reaction, a slide projector is used as a light sourceCorrect Choice B. Repeated MEDs to UVB or UVA, given to different test sites over several days can be used to reproduce lesions of polymorphous light eruption C. The UVA MED is performed using a narrowband UVA source D. A single large dose of visible light is the best way to reproduce lesions of polymorphous light eruption E. The UVB MED is performed using narrowband UVB light sources Phototesting is done prior to initiating phototherapy or during provocative induction. To test for a visible light reaction, a slide projector is used as a light source. The other choices are incorrect. The UVB MED and UVA MED is performed using BROADBAND UVB and BROADBAND UVA light sources respectively. Repeated MED's to UVB or UVA given to the SAME site may be used to reproduce lesions of PMLE. Alternatively a single large dose of UVA or UVB can be used to reproduce PMLE 7) The main contributor to erythema of the skin with exposure to the sun is: A. UVC B. Visible light C. UVA2 D. UVA1 E. UVBCorrect Choice UVB in natural sunlight is the main contributor to erythema. UVB erythema reaches a maximum in 6-24 hours. UVA accounts for 15-20% of sunlight erythema despite that there is much more UVA than UVB in sunlight 8) Treatment of CAD might include: A. All of these answers are correctCorrect Choice B. PUVA C. UV filters for car windows 2
  • 3. D. Incandescent bulbs for home and office lighting E. Broad spectrum sunblock to which the patient is not allergic CAD patients are exquisitely sensitive to UVB, often sensitive to UVA, and sometimes sensitive to visible light. Answers a, b, and c are correct because those treatments prevent UV and visible radiation from reaching the patient. PUVA hardens and desensitizes the skin 9) The xenon arc solar simulator, whose spectrum is in this slide, is useful for: A. MED-B testing and MED-A testing Correct Choice B. MED-A testing C. Photopatch testing D. MED-B testing, MED-A testing, and Photopatch testing E. MED-B testing Solar simulator radiation contains both UVB and UVA in sufficient quantities to be useful for MED-B and MED-A testing. The field size is too small for photopatch testing 10) Rare cases of which photosensitivity disorder have been associated with erythropoietic protoporphyria? A. Solar urticariaCorrect Choice B. Chronic actinic dermatitis C. Actinic prurigo D. Hydroa vacciniforme E. Polymorphous light eruption Rare cases of solar urticaria have been associated with erythropoietic protoporphyria (EPP), lupus erythematosus (LE), and with certain drugs. Blood tests for LE and EPP should be performed in patients with this diagnosis 11) Fluorescent UVA bulbs used for phototesting or PUVA therapy have a peak emission at: A. 254 nm B. 468 nm C. 311 nm D. 312 nm E. 352 nmCorrect Choice 254 nm is the wavelength of the radiation emitted by mercury vapor lamps. Narrowband UVB emits 311-312 nm. Fluorescent UVA bulbs used for phototesting or PUVA therapy have a peak emission at 352 nm 12) A patient that rarely burns and usually tans is which of the following skin types? A. Type IV B. Type IIICorrect Choice C. Type I 3
  • 4. D. Type V E. Type II Skin type I always burns and never tans Skin type II usually burns and rarely tans Skin type III rarely burns and usually tans Skin type IV never burns and alway tans Skin types V,VI are highly pigmented individuals 13) The wavelength range that most effectively induces cutaneous immunosuppression is: A. 400-410nm B. 320-340nm C. 290-320nmCorrect Choice D. 340-400nm E. 200-290nm 290-320nm (UVB) is most effective in suppressing cutaneous immunity; mechanisms include depletion of Langerhans cells, induction of regulatory T cells, and keratinocyte secretion of such immunosuppressive cytokines as IL-10 and TNF-alph 14) Which of the following statements is true regarding pseudoporphyria? A. None of these answers are correct B. It has been associated with furosemide C. All of these answers are correctCorrect Choice D. Clinically it may be indistinguishable from porphyria cutanea tarda E. Porphyrins may be normal Pseudoporphyria is a phototoxic reaction that clinically and histologically resembles porphyria cutanea tarda. In pseudoporphyria, however, there is no porphyrin abnormality. It has been associated with numerous medications, including nalidixic acid, tetracyclines, amiodarone, furosemide, and ketoprofen 15) Oral PUVA has been used to treat which of the following diseases: A. All of these answers are correctCorrect Choice B. Graft-versus-host disease C. CTCL D. Mastocytosis E. Generalized granuloma annulare PUVA has been reported to have helped patients with all the diseases listed 16) Regarding renal transplant recipients, which of the following statements is correct: A. Should have skin cancer screening at the same rate as the general population B. Have a 36-fold increased risk of BCC 4
  • 5. C. Have rates of SCC similar to the general population D. Have an decreased risk for melanoma E. Have a 36-fold increased risk of SCCCorrect Choice The rates of SCC in renal cell transplant recipients is increased at a rate of 36x. The rates of skin cancer are higher than in the general population 17) The usual dose of UVR given for photopatch testing is approximately: A. 10 Joules of UVA B. 4 millijoules of UVB C. 4 Joules of UVB D. 2 Joules of UVA Correct Choice E. 10 Joules of UVB UVA is absorbed by the chemicals that cause photoallergic contact dermatitis; therefore, it is UVA that is used for photopatch testing. In patients who are UVA-sensitive, one-half of their MEDA is used for photopatch testing 18) A patient presents with signs of porphyria cutanea tarda. Porphyrin screens are negative. Which of the following medications on the patient's medication list is your top choice for discontinuation? A. Fenofibrate B. Multivitamin C. NaproxenCorrect Choice D. Chloroquine E. Enalapril NSAIDs are a frequent offender in causing pseudoporphyria. The other listed medications are not frequent causes of this type of skin reaction 19) The typical patients with polymorphous light eruption are: A. Fair-skinned females in their teens and 20’s. Correct Choice B. Type V-skinned females in their 20’s and 30’s. C. Fair-skinned males in their 20’s and 30’s. D. Fair-skinned females in their 40’s and 50’s. E. Type IV-skinned males in their 40’s and 50’s. PMLE most commonly appears in fair-skinned females during the first three decades of life. It may be related to type IV hypersensitivity 20) All of the following statements regarding chronic actinic dermatitis are true EXCEPT: A. Azathioprine has been used successfully to treat this disorder B. The MEDB on phototesting is markedly diminished in patients with this condition C. Many cases begin as photoallergic contact dermatitis or drug photosensitivity 5
  • 6. D. Fluorescent bulbs are safer for these patients than are incandescent bulbsCorrect Choice E. Many patients have a lowered threshold to shorter wavelength visible light Chronic actinic dermatitis (CAD) usually occurs in middle-aged to elderly males who present with a chronic, eczematous dermatitis in a photodistribution, though there is no history of current exposure to a photosensitizer. Phototesting is very helpful in diagnosing CAD. The MEDB is markedly diminished, and the MEDB site may show an eczematous or infiltrated appearance. Many of the patients have a lowered MEDA as well, and may have a lowered threshold to shorter wavelength visible light in the blue-violet end of the spectrum. Many cases of this idiopathic disorder are thought to have begun as photoallergic contact dermatitis or as a drug photosensitivity with broadening of the photosensitivity to include the UVB range. It is unclear why photosensitivity persists when the photosensitizer is no longer present. Treatment includes strict sun avoidance and sun protection. Incandescent bulbs with longer wavelengths, far from the blue-violet end of the visible spectrum, should be used instead of fluorescent bulbs, which have significant blue-violet radiation. Topical and oral steroids, oral azathioprine and cyclosporine, and PUVA have all been used to treat patients with CAD 21) Treatment of polymorphous light eruption includes all of the following EXCEPT: A. None of these answers are correctCorrect Choice B. PUVA therapy C. Systemic corticosteroids D. Antimalarials E. Topical steroids Most patients with PMLE have mild disease that can be treated by sun avoidance and sun protection. Topical steroids can be used to treat clinical lesions. For severe cases, hardening and desensitization can be accomplished with UVB, UVB plus UVA, or PUVA. Antimalarials can be used for resistant cases. A short course of prednisone (20-40 mg) is effective for brief, sunny vacations 22) Which of the following statements about ultraviolet C is incorrect? A. UV-C is absorbed by atmospheric ozone B. UV-C has an electromagnetic spectrum from 200-290 nm C. UV-C has a higher wavelength than UV-BCorrect Choice D. UV-C has higher energy than UV-B E. UV-C does not reach the earth's surface UV-C has wavelengths of 200 - 290 nm. UV-B has wavelengths of 290 - 320 nm. UV-C has a lower wavelength, not higher. All of the other listed statements about UV-C are correct 23) Which of the following statements about the spectrums of UVR that cause of solar urticaria is MOST correct? A. Visible light causes solar urticaria B. UVA causes solar urticaria C. UVB causes solar urticaria D. Both UVA and UVB cause solar urticaria E. Patients can react to visible light, UVA and/or UVBCorrect Choice 6
  • 7. Some patients react with wheals to either visible light or UVA or UVB. Others react to both UVA and visible radiation. Some react to both UVB and UVA, and some patients react to UVB, UVA and visible light 24) Patients with chronic actinic dermatitis (CAD) typically show on phototesting: A. Lowered MEDB, usually normal MEDA, sometimes positive photopatch tests B. Lowered MEDB, usually lowered MEDA, sometimes positive photopatch tests Correct Choice C. Normal MEDB, lowered MEDA, sometimes positive photopatch tests D. Normal MEDB, normal MEDA, sometimes positive photopatch tests E. None of the above The hallmark of CAD is a lowered MEDB. Often the patients have a lowered MEDA. It is felt that many of CAD patients began with photoallergic contact dermatitis, so some have positive photopatch tests 25) Most patients with PMLE require treatment with: A. Sunscreen and sun avoidance between 11 am and 3pmCorrect Choice B. Cyclophosphamide C. Chloroquine D. UVB hardening/desensitization E. Prednisone Most patients have mild disease that can be treated by sun avoidance, especially between 11 am and 3pm; a broad spectum sunscreen and clothing with a tight weave. In more severe cases, UV hardening, antimalarials or prednisone can be used. Cyclophosphamide is not used in PMLE 26) The histology of the patient in question 42 is the following slide. It mimics the histology of: A. CTCL Correct Choice B. Pityriasis rosea C. Psoriasis D. Tuberculoid leprosy E. SLE The histology of the actinic reticuloid variety of CAD mimics that of mycosis fungoides with a band- like infiltrate with atypical mononuclear cells 27) The highest energy visible photons are in which portion of the visible spectrum? A. All of these answers are correct B. Green C. Yellow D. Blue-violet Correct Choice E. Red-Orange 7
  • 8. In the visible spectrum, the blue-violet portion has the shortest wavelength and the highest energy. The red-orange portion has the longest wavelength and the lowest energy 28) In solar urticaria wheals typically: A. Begin at 1-2 h. and last 2-4 h B. Begin at 15-30 minutes and last 1 h Correct Choice C. Begin at 45 minutes after exposure and last 2 hours (h.) D. Begin at 15-30 seconds and last about 15 minutes E. Begin at 30-60 minutes and last 1-4 h The wheals of solar urticaria begin 10-15 minutes after exposure and last for about an hour. Solar urticaria is usually idiopathic. Rare cases are associated with EPP, SLE, and certain medications 29) Currently the most common cause of photoallergic contact dermatitis is: A. Musk ambrette B. Halogenated salicylanilides C. Mercaptobenzothiazole D. 6-methylcoumarin E. Sunscreens Correct Choice Halogenated salicylanilides, musck ambrette and 6-methylcoumarin used to cause most of the cases of photoallergic contact dermatitis. They have been essentially eliminated from soaps and fragrances. Sunscreen ingredients are now the most common cause 30) The potent photosensitizer, 5-methoxypsoralen, is contained in which of the following contact allergens? A. Eugenol B. Oil of BergamotCorrect Choice C. Usnic acid D. Balsam of Peru E. Tuliposide A Oil of Bergamot contains 5-methoxypsoralen and may cause of 'Berloque dermatitis,' an intense post-inflammatory hyperpigmentation due to an antecedent phytophotodermatitis that may be subclinical 31) A common cause of medication induced photoallergy is: A. Naproxen B. Acitretin C. Psoralens D. PiroxicamCorrect Choice E. Ibuprofen 8
  • 9. All of the listed medications are causes of photosensitivity, but only piroxicam is a cause of photoallergy 32) Phototesting of PMLE patients reveals: A. Normal MEDB, reduced MEDA. B. Normal MEDB, elevated MEDA. C. Normal MEDB, normal MEDA. Correct Choice D. Lowered MEDB, normal MEDA. E. Lowered MEDB, lowered MEDA. The MEDB and MEDA are normal in PMLE patients. Only with multiples of the MEDB or MEDA can one often reproduce the lesions 33) Ultraviolet radiation from the sun cause which of the following acute effects EXCEPT? A. Increased immune surveillance Correct Choice B. Immediate pigment darkening C. Epidermal thickening D. Sunburning E. Photosynthesis of vitamin D UV radiation causes acute effects including: photosynthesis of vitamin D, sunburning, immediate pigment darkening, delayed tanning, epidermal thickening and immunologic effects such as DECREASED immune surveillance 34) A 40-year-old patient presents with widespread plaque-type psoriasis without arthritis. She had already applied potent topical steroids, calcipotriene, and tazarotene. The treatment of choice would be: A. Methotrexate B. Narrowband UVB Correct Choice C. Etretinate D. Cyclosporine E. PUVA The other choices involve internal medications and therefore have the potential for more side effects than with narrowband UVB. Phototherapy may be impractical for some patients, and therefore systemic therapies should be offered 35) Which of the following is true regarding actinic prurigo? A. Lesions generally continue through late adulthood B. Cheilitis is frequently seenCorrect Choice C. Thalidomide has been ineffective for the majority of patients D. Lesions persist for 1-2 days E. Lesions never occur on non-sun-exposed areas 9
  • 10. Actinic prurigo is an idiopathic photosensitivity disorder. Lesions are excoriated papules and nodules that begin in childhood and remit in puberty. They can last for several months and may occur on non-sun-exposed areas. Thalidomide has been very effective for treating the majority of patients with actinic prurigo. 36) The irradiance of a UV source is measured in: A. Joules B. Seconds C. Millijoules D. Centimeters E. Watts Correct Choice The irradiance of a UV source in watts x the time in seconds equals the UV dose in joules 37) A patient presents with solar urticaria. What tests should be considered initially? A. ESR B. ANACorrect Choice C. CXR D. ANCA E. Urinalysis Solar urticaria is an idiopathic, type I photosensitivity disorder. Rare cases have been associated with erythropoietic protoporphyria (EPP) and lupus erythematosus (LE). Blood tests for LE and appropriate screening for EPP should be performed including ANA, Ro/La, Urine & Stool porphyrins 38) The best location for phototesting patients suspected of photosensitivity is: A. Unaffected skin of the upper back B. Effected skin of the ventral forearm C. Effected skin of the buttock D. Unaffected skin of the lower backCorrect Choice E. Unaffected skin of the outer thighs Patients with suspected photosensitivity can be tested on unaffected skin of the buttocks, lower back or ventral forearm. Effected skin should not be used for testing 39) Blue-gray pigmentation of the lunulae: A. Is a rare side effect of treatment with tricyclic antidepressants B. None of these answers are correct C. Occurs only after many years of treatment with chlorpromazine D. Is seen in argyria and not in most medication photosensitivitiesCorrect Choice E. Can be seen in patients on amiodarone Blue-gray pigmentation on sun-exposed areas can be seen during treatment with amiodarone, chlorpromazine, and tricyclic antidepressants. In these medication photosensitivities, there is no 10
  • 11. involvement of the sclera, lunulae, or mucous membranes, as in argyria (prolonged contact with or ingestion of silver salts 40) The best definition for a MED is: A. The dose of ultraviolet radiation that produces pronounced erythema that completely fills the test square B. The dose of ultraviolet radiation that is one step below the first visible erythema C. The dose of ultraviolet radiation that produces bullae that completely fills the test square D. The dose of ultraviolet radiation that produces a easily visible "sunburn" E. The dose of ultraviolet radiation that produces barely perceptible erythema that completely fills the test squareCorrect Choice An MED is the dose of ultraviolet radiation that produces barely perceptible erythema that completely fills the test square. The remaining options are incorrect 41) The image shown is diagnostic of which of the following conditions? A. Bullous pemphigoid B. Pemphigus vulgaris C. Chronic actinic dermatitis D. Polymorphous light eruptionCorrect Choice E. Pseudoporphria cutanea tarda The pathologic image shown is that of polymorphous light eruption. The histology shows a lymphocytic infiltrate around the superficial vascular plexuses with subepidermal edema and minimal epidermal changes are present 42) Photoexacerbated genodermatoses include all except: A. Hartnup Disease B. Darier's Disease C. Hailey-Hailey Disease D. Cockayne's Syndrome E. Tuberous sclerosis Correct Choice Tuberous sclerosis is not associated with photosensitivity 43) Which of the following statements is correct regarding antigen presenting cells after UVR exposure? A. Have increased ability to prime UV-irradiated mice to subcutaneously injected hapten B. UV-irradiated mice have normal antigen presentation, allowing a normal delayed-type hypersensitivity response C. APC's have increased ability to prime UV-irradiated mice to subcutaneously injected protein D. APC's have increased ability to prime UV-irradiated mice to applied contact-sensitizing agents E. There is a decrease in the number of antigen presenting cellsCorrect Choice 11
  • 12. APC's have depressed ability to prime UV-irradiated mice to subcutaneously injected hapten or protein and to applied contact-sensitizing agents. UV-irradiated mice have defective antigen presentation, preventing a normal delayed-type hypersensitivity response. There is a REDUCTION in number of antigen presenting cells 44) The UVAII portion of the electromagnetic spectrum extends from: A. 290-320 nm B. 320-340 nmCorrect Choice C. 320-400 nm D. None of these answers are correct E. 200-290 nm UVA extends from 320-400nm. UVA is further subdivided into UVAII (320-340nm) and UVA1 (340-400nm 45) Which of the following statements about UVR and DNA is correct? A. Pyrimidine dimers can not activate oncogenes B. UVA is most effective in producing pyrimidine dimer C. UVA is much more efficient than UVB in inducing DNA damage D. Cells from patients with actinic keratoses have more DNA repair capacity than controls E. UVR alters DNACorrect Choice UVR alters DNA. UVB is much more efficient than UVA in inducing DNA damage. It can create pyrimidine dimers that may activate oncogenes and lead to cancer. Cells from patients with actinic keratoses have less DNA repair capacity than controls 46) Ultraviolet light spectrum most completely encompasses which of the following spectrums? A. 760 - 1200 nm B. 290 - 400 nm C. 10 - 400 nmCorrect Choice D. 400 - 700 nm E. 200 - 400 nm The ultraviolet spectrum encompasses Vacuum UV from 10-200nm, UV-C from 200-290nm, UV-B from 290-320nm and UV-A from 320-400nm. The visible light spectrum is from 400-700nm. 700-1200 nm is part of the infrared spectrum 47) Which of the following is true about UV light: A. Clouds absorb most UVA light B. Sunlight early in the morning and late in the day contains relatively more UVB C. UVB radiation is 1000 times more erythrogenic than UVACorrect Choice D. UVAII light is 340-400nm E. UVA radiation is 1000 times greater than UVB during midday hours 12
  • 13. UVA light is found b/w 320 and 400nm and is broken up into UVAI(340-400nm) and UVAII((320-340nm). UVB light is found between 290 and 320nm. UVA radiation is 100 times greater than UVB during midday hours and sunlight early in the morning and late in the day contains relatively more UVA. UVB radiation is 1000 times more erythrogenic than UVA. Cloud cover is a poor UV absorber 48) Which of the following hypoglycemics is the most common cause of photosensitivity? A. Thiazolidinediones (i.e. rosiglitazone) B. Metformin C. Insulin D. Piroxicam E. SulfonylureasCorrect Choice The sulfonylurea hypoglycemics for diabetes are the most common type of medication of this class of medications. Piroxicam is not a hypoglycemic agent, but a common NSAID cause of photoallergy 49) The most helpful phototest to document this photosensitivity disorder would be: A. MEDBB B. Repeated doses of UVA and UVB Correct Choice C. Photopatch tests D. MEDNB E. All of these answers are correct Repeated doses of ultraviolet radiation can sometimes elicit lesions of PMLE. More patients react to UVA radiation than to UVB radiation 50) Hereditary PMLE of Native Americans: A. Often presents with chelitis and conjunctivitisCorrect Choice B. Rarely persists into adulthood C. All of these answers are correct D. Is not treated with thalidomide E. Is more similar to PMLE than to actinic prurigo Hereditary PMLE of Native Americans is similar to actinic prurigo but persists much more frequently into adulthood. 75% of patients have a positive family history. It presents with a papular, excoriated, eczematous dermatitis that occurs predominantly on the face. Chelitis and conjunctivitis are common. It may be treated with thalidomide 51) Solar urticaria: A. Rarely lasts for more than 6 months to 1 year B. Is not benefitted by antihistamines C. Can present with headache, nausea, and syncopeCorrect Choice D. Is an idiopathic, type IV photosensitivity disorder 13
  • 14. E. Usually occurs to only UVB radiation Solar urticaria is an idiopathic, type I photosensitivity disorder. Mediator release during widespread whealing may result in headache, nausea, wheezing, faintness, and syncope. It usually lasts for many years. Some patients react with wheals to either visible light or UVA or UVB. Others react to both UVA and visible radiation. Some react to both UVB and UVA, and some patients react to UVB, UVA, and visible radiation. Treatment includes sun avoidance and protection, and H-1 antihistamines may be of partial benefit 52) Most bulbs used for PUVA have a peak output predominantly in the following range: A. 320 nm – 340 nm B. 390 nm – 410 nm C. 340 nm – 400 nm D. 350 nm – 360 nm Correct Choice E. 290 nm – 320 nm The UVA emitted by these bulbs is absorbed by psoralens, causing covalent bonding of psoralens to DNA 53) The active spectrum for cutaneous vitamin D3 synthesis is: A. 320-400nm B. 400-410nm C. 290-320nmCorrect Choice D. 410-450nm E. 220-290nm The source states that vitamin D3 synthesis occurs at wavelengths < 320 making choices 220-290nm and 290-320nm correct 54) When solar urticaria is a consideration for phototesting: A. 7 test squares of increasing UV doses should be exposed B. An MED (B) and MED (A) should not be performed C. The lower back should not be used for testing D. Visible light will not evoke the lesions E. An additional reading at 15 minutes after exposure should be performedCorrect Choice When solar urticaria is a consideration, an additional reading at 15 minutes after exposure is important, as wheals begin within 10-30 minutes after exposure and last for about one hour. The face and hands may not show lesions as they are chronically exposed to sun. Some patients react to either visible light or UVA or UVB. Others react to both UVB and UVA, both UVA and visible light, or all three 55) Which of the following statements regarding hereditary PMLE of Native Americans is true? A. Specific HLA types predominate in Native AmericansCorrect Choice B. Cheilits and conjunctivitis are uncommon 14
  • 15. C. 5-10% of patients have a family history of this eruption D. It will remit in puberty E. Patients have an urticarial erupion. Native Americans have a high rate of PMLE and there is some evidence of a genetic predisposition. It tends to have a specific HLA predominance, continues through adulthood, and commonly presents as a papular, excoriated, eczematous dermatitis predominantly on the face. Cheilitis and conjunctivitis are common. Up to 75% of patients have a positive family history 56) The portion of the electromagnetic spectrum that produces a particular biologic effect is known as the: A. Photobiologic spectrum B. Minimal erythema dose C. Active spectrum D. Action spectrumCorrect Choice E. Absorption spectrum The action spectrum is the portion of the electromagnetic spectrum that products a particular biologic effect (e.g. erythema, delayed tanning). The absorption spectrum is the portion of the electromagnetic spectrum that is absorbed by a particular absorbing molecule, or chromophore. The minimal erythema dose is the dose of UV radiatino that produces barely perceptible erythema during phototesting 57) A patient presents with onycholysis after sun exposure. Which of the following medications would you rule out as a cause of this presentation? A. Quinolones B. Quinine C. ChlorpromazineCorrect Choice D. Tetracyclines E. Psoralens Chlorpromazine is associated with blue-gray pigmentation on sun-exposed areas and is not associated with photoonycholysis. Quinolones, tetracyclines, psoralens and quinine can cause photoonycholysis 58) Phytophotodermatitis can be seen with the following fruits / vegetables: A. Potatoes and leeks B. Tomatoes and bananas C. Celery and radishes D. Parsnips and limesCorrect Choice E. Lemons and pears Psoralens in certain plants, fruits, and vegetables can cause phytophotodermatitis. The most common ones are limes, figs, parsley, parsnip, bergamot oranges, and celery 59) Narrow band UVB consists of what wavelength? 15
  • 16. A. 308-310nm B. 311-313nmCorrect Choice C. 300-306nm D. 312-320nm E. 320-330nm Narrowband UVB is much less erythemogenic with regard to physical units (mJ/cm2) than broadband UVB. Narrowband UVB is 311-313nm 60) Absorption of UV radiation generates singlet oxygen in the skin by which chromophore? A. Melanin B. Water C. Hemoglobin D. Keratin E. Urocanic acidCorrect Choice Urocanic acid and DNA are biologically important chromophores. DNA absocrbs uVB directly inducing changes between adjacent pyrimidine bases on one strand of DNA. Cyclopyrimidine dimers, particularly thymine dimers or less commonly (6-4) photoproducts may be generated. Urocanic acid is a second biologically important chromophore in the skin and is a by-product of filaggrin breakdown. One photon of light contains enough energy to generate singlet oxygen. 61) Which of the following cell types induce susceptibility to tumor growth? A. NK cells B. Langerhans cells C. Helper T-cells D. Suppressor T-cellsCorrect Choice E. Mast cells The suppressor T-cells induce susceptibility to tumor. These cells appear to arise in UV-irradiated hosts prior to tumor developing, and play a role in carcinogenesis 62) Possible treatments for this patient with vitiligo include all of the following except: A. Topical steroids B. AcitretinCorrect Choice C. Narrowband UVB D. Oral PUVA E. Broadband UVB Topical steroids and all the UV choices have been used to treat vitiligo. Soriatane does not benefit vitiligo 63) The solar simulator is not useful for phototherapy because of its: A. Low output of UVB 16
  • 17. B. High output of infrared radiation C. Small field sizeCorrect Choice D. Low level of ionizing radiation E. High output of ultraviolet radiation The coin-sized field of the solar simulator prevents this source from being useful for phototherapy. It is useful in MED testing 64) All of the following are true regarding actinic reticuloid except: A. Is a premalignant conditionCorrect Choice B. CD8+ T cells in lesional skin C. Generalized lymphadenopathy common D. Atypical dermal mononuclear cell infiltrate E. Affects elderly men Actinic reticuloid is a type of chronic actinic dermatitis. Ive et al. introduced the disease as a severe dermatosis with no apparent photoallergen. It generally affects elderly males and is characterized by infiltrated erythematous plaques on an eczematous background in exposed sites with lymphadenopathy. Histopathologically, it may resemble cutaneous T cell lymphoma. However, there is a trend towards a lower CD4+/CD8+ ratio. It is not considered a premalignant condition 65) Which medication reactivates UVB- and PUVA-induced erythema? A. MethotrexateCorrect Choice B. 5-FU C. None of these answers are correct D. Dacarbazine E. Vinblastine Methotrexate reactivates UVB- and PUVA-induced erythema 66) The patient demonstrates a positive photopatch test to musk ambrette and a lowered MEDB. The correct diagnosis is most likely: A. Solar urticaria B. Photoallergic contact dermatitis C. CAD Correct Choice D. Actinic prurigo E. PMLE Only chronic actinic dermatitis (CAD) has a lowered MEDB. That finding helps distinguish CAD from photoallergic contact dermatitis in which one sees a positive photopatch test also 67) A joule is a measurement of: A. Wavelength B. Power 17
  • 18. C. EnergyCorrect Choice D. Irradiance E. Distance A joule is a measurement of energy dose or fluence. Power or irradiance is measured in watts. These are related by the formula Joules/cm2=Watts/cm2xseconds 68) What is the most likely cause of these lesions on the cheek of this 8 year-old boy? A. Varicella B. Herpes simplex C. Polymorphous light eruption D. Acne E. Hydroa vacciniformeCorrect Choice Hydroa vacciniforme is a rare photodermatosis of childhood which occurs on areas of sun-exposed skin. The lesions leave depressed and atrophic scars. The condition tends to spontaneously resolve in adulthood 69) Lesions of PMLE typically appear: A. Immediately B. Hours to days after exposureCorrect Choice C. About one hour after exposure D. Days to two weeks after exposure E. 15-30 minutes after exposure The history of a delay of several hours to several days after exposure is important to the diagnosis 70) The following compound exerts immunosuppressive effects in the skin following exposure to UV-radiation: A. Cis-urocanic acidCorrect Choice B. Delta aminolevulinic acid C. IL-12 D. Amino-levulinic acid E. 7-dehydrocholesterol Trans-urocanic acid is an epidermal chromophore that isomerizes to cis-urocanic acid following exposure to UV radiation. Cis-urocanic acid has been shown to be immunosuppressive, for example, by impairing the induction of contact allergy in mouse models. The mechanism of this immunosuppressive effect is unclear 71) UVA II encompasses which wavelengths ? A. 290-320 nm B. 340-400 nm C. 400-450 nm 18
  • 19. D. 320-340 nm Correct Choice E. 320-400 nm UVA can be divided into UVA II (320-340 nm) and UVA I (340-400 nm) 72) In solar urticaria wheals may be brought on by: A. UVA, UVB and Visible light Correct Choice B. UVB C. UVC D. UVA E. Visible light Some patients react to visible light or UVA or UVB. Others react to combinations of visible light, UVA, and UVB 73) All of the following are true about UVA radiation except: A. penetrates to a greater depth in the dermis than UVB B. virtually all blocked by car window glassCorrect Choice C. responsible for phototoxic drug reactions D. 10 times more abundant than UVB E. approximately 50% of exposure occurs in the shade The UVA band extends from 320 to 400 nm. This spectrum is further subdivided into UVA-2 (320 to 340 nm) and UVA-1 (340 to 400 nm). The UVA spectrum is recognized as a cause of immediate and delayed tanning reaction of skin, and several other effects including photoaging, skin photosensitization, and immunosuppression. The majority of the ultraviolet radiation at the earth's surface is UVA (95 to 98%) with only 2 to 5% comprised of UVB. As UVC is completely absorbed by the stratospheric ozone layer, it does not comprise ultraviolet radiation hitting the earth's surface. Much of the UV radiation after reaching the atmosphere becomes scattered by the time it hits the earth's surface. Due to this "sky radiation", it is possible to sunburn even if one is exposed only to the shade. Notably, window glass filters out ultraviolet wavelengths shorter than 320 nm, so only UVB (290 to 320 nm) and UVC (200 to 290 nm) are effectively filtered by car window glass. Although UVA penetrates deeper into the dermis than UVB, UVB radiation is much more erythmogenic. Finally, most common photosensitizers have action spectrums in the UVA range, and, as a result, UVA radiation is responsible for most phototoxic drug reactions 74) This malnourished individual presented with crusting and hyperpigmentation in a photodistribution. The best diagnosis is: A. PMLE B. Photoallergic contact dermatitis C. Scurvy D. CAD E. Pellagra Correct Choice Pellagra is characterized by the triad of diarrhea, dermatitis, and dementia. The dermatitis begins as a burning erythema in sun-exposed areas. There may be bullae and erosions. This is followed by a dry, brittle, scaling and hyperpigmented phase. Pellagra is due to a deficiency of niacin and tryptophan 19
  • 20. 75) Medication photosensitivity is caused by all except: A. Cephalosporins Correct Choice B. Thiazides C. Phenothiazenes D. Doxycycline E. Quinolones Cephalosporins do not cause drug photosensitivity. Neither do the penicillins 76) Which of the following statements about electromagnetic radiation is MOST correct? A. Electromagnetic radiation is measured in watts B. The energy of photons is proportional to the wavelength C. The energy of photons is inversely proportional to the frequency D. Longer wavelengths penetrate the skin more deeplyCorrect Choice E. Electromagnetic radiation can be conceptualized as packets of power called photons Longer wavelengths penetrate the skin more deeply. Electromagnetic radiation can be conceptualized as packets of ENERGY called photons. The energy of photons is proportional to the FREQUENCY and inversely proportional to WAVELENGTH. Electromagnetic radiation is measured in wavelength 77) A MED phototest should be read at: A. 24 hoursCorrect Choice B. 12 hours C. 48 hours D. 96 hours E. 2 hours MED testing should be read 24 hours after delivery of the doses. An additional reading at 15 minutes is important when solar urticaria is a consideration 78) Lichen planus-like lesions on sun-exposed areas may be seen in patients being treated with: A. None of these answers are correct B. All of these answers are correct C. Quinolones D. Ketoprofen E. FenofibrateCorrect Choice Lichenoid medication photosensitivity, with LP-like lesions on sun-exposed areas has been associated with treatment with antimalarials, thiazides, demethylchlortetracycline, fenofibrate, enalapril, quinine, and quinidine 79) Photoonycholysis has been attributed to: 20
  • 21. A. Tricyclic antidepressants B. QuinineCorrect Choice C. Furosemide D. Chlorpromazine E. Amiodarone Photoonycholysis is a manifestation of medication photosensitivity that has been attributed to quinolones, tetracyclines, psoralens, and quinine 80) The immunologic effects of UVR include all of the following except: A. Decrease in IL-1, IL-6, TNF-á Correct Choice B. Alteration of lymphocyte population C. Suppression of delayed-type hypersensitivity D. Alteration of Langerhans cell function E. Alteration of Langerhans cell morphology UVR causes an increase of circulating cytokines (IL-1, IL-6, TNF-a). 81) Regarding the UVR effects on contact dermatitis and delayed-type hypersensitivity, which of the following statements is correct? A. There are increases in production of Th2 type cytokines B. There are increased delayed-type hypersensitivity responses C. Induction of sensitization is increased D. Mice exposed to long-term, high-dose UVR demonstrate increased splenic APC function E. There are diminished contact hypersensitivity responsesCorrect Choice Mice exposed to short-term, high-dose UVR demonstrate decreased splenic APC function. There are diminished delayed-type hypersensitivity and contact hypersensitivity responses. Induction of sensitization is decreased. Th2 cytokines are not increased following UVR exposure 82) This patient’s MEDB was less than 1 mJ/cm2. His medications were Lipitor, digoxin, and Coumadin. The most likely diagnosis is: A. Photoallergic contact dermatitis B. SLE C. CAD Correct Choice D. PMLE tested E. Drug photosensitivity CAD is the most likely cause of such a low MEDB. Drug photosensitivity can show a lowered MEDA and on occasion a lowered MEDB, but none of the patient’s drugs cause photosensitivity 83) Phototoxic reactions: A. Rarely occur on the first exposure to the chemical 21
  • 22. B. Resolve with hyperpigmentationCorrect Choice C. Occur only in predisposed individuals D. Are immunologically mediated E. Are called "photoreactive" if they produce damage through reactive oxygen species A phototoxic reaction appears as a exaggerated sunburn with erythema and sometimes blistering, resolving with hyperpigmentation. It is a nonimmunologic reaction that could occur in all individuals given enough of the chemical and enough UVR. It can occur on the first exposure to the chemical and the UVR. Phototoxic reactions that produce damage through reactive oxygen species are called "photodynamic." 84) Initial treatment of this disease should include: A. Hydroxychloroquine B. Sun avoidance, sunblocks, desensitization C. Car UV filters, PUVA, sunblocks D. Sun avoidance, sunblocks, beta carotene E. Sun avoidance, sunblock, antihistamines Correct Choice Treatment of solar urticaria is difficult. Sun avoidance is the most important aspect of treatment. A broad-spectrum sunblock and antihistamines 85) UVB converts 7-dehydrocholesterol in the skin to: A. Previtamin D3Correct Choice B. Calcitriol C. 1,25-dihydroxyvitamin D D. None of the above E. 25-hydroxyvitamin D UVB converts 7-dehydrocholesterol in the skin to previtamin D3, which then thermally isomerizes to form vitamin D3. It is hydroxylated in the liver and then in the kidney to form 25-hydroxyvitamin D and 1,25-dihydroxyvitamin D, respectively 86) Lichen planus like lesions on sun-exposed areas may be seen in patients receiving which medication? A. Alprazolam B. Griseofulvin C. FenofibrateCorrect Choice D. All of these answers are correct E. None of these answers are correct LP-like lesions (which may be confluent) on sun-exposed areas have been seen in patients receiving antimalarials, thiazides, demethylchlortetracycline, fenofibrate, enalapril, quinine, and quinidine 87) Common side effects of PUVA include all of the following except: 22
  • 23. A. Painful erythema B. Squamous cell carcinoma C. Hair loss Correct Choice D. Prolonged pruritus E. Nausea Alopecia is not a usual side effect of PUVA treatment 88) Which of the following statements about the light sources for phototesting/phototherapy is correct? A. The mercury vapor is excited by electric current and emits a line spectrum of 254 nmCorrect Choice B. Phototherapy bulbs are low-pressure sulfur vapor lamps with the inner surface coated by a specific phosphor C. The phosphor emits a discoherent spectrum of various wavelengths D. The most common light sources are incandescent bulbs E. Broadband UVB bulbs emit throughout the UVB range and also include some UVC Fluorescent bulbs are commonly used for phototherapy. These bulbs are low-pressure mercury vapor lamps with the inner surface coated by a specific phosphor. The phosphor emits a continuous spectrum of various wavelengths. The mercury vapor is excited by electric current and emits a line spectrum of 254 nm. Broadband UVB bulbs emit throughout the UVB range and also include some UVA, not UVC. 89) Lumisterol is an inactive epidermal reservoir of which vitamin? A. E B. K C. A D. DCorrect Choice E. C Lumisterol and tachysterol are inert byproducts created during the biosynthesis of vitamin D. When Previtamin D3 is exposed to light, it can result in photoisomerization of previtamin D3 to lumisterol and tachysterol. If previatmin D3 is depleted, lumisterol and tachysterol can become converted back to previtamin D3 90) Oxsoralen plus UVA results in the following except: A. Has immunomodulating effects B. Suppresses DNA synthesis C. Forms monofunctional adducts D. Binds to purine bases Correct Choice E. Can form DNA crosslinks Oxsoralen, in the presence of UVA, forms covalent bonds to pyrimidine bases on DNA 23
  • 24. 91) Advantages of narrowband UVB over PUVA therapy are the following except: A. No nausea B. Safe in pregnancy C. No need for protective eyewear D. Safe in childhood E. More effective in treating thick plaques of CTCL Correct Choice UVB, both narrowband and broadband, is less penetrating into the skin than UVA. Therefore, PUVA is more effective for thick plaques of CTCL. The UVB does not reach to the bottom of the plaques 92) This middle-aged man demonstrates infiltrated, dusky plaques on all sun-exposed areas. The most likely diagnosis is: A. Actinic prurigo B. CAD Correct Choice C. Drug photosensitivity D. Photoallergic contact dermatitis E. PMLE The thick, infiltrated plaques on sun-exposed areas are typical of the actinic reticuloid variety of CAD 93) Which of the following statements about the hypothetical effect of UVR on cancer induction is correct? A. UVR alters APC function, by increasing the number of antigen-presenting cells B. Suppressor T-cells are suppressed C. UVR induces transformation of keratinocytes with expression of tumor-associated antigensCorrect Choice D. UVR increases the ability of Langerhans cells to present antigen E. UVR inhibits the release of immunosuppressive factors UVR induces transformation of keratinocytes with expression of tumor associated antigens. It alters APC function by reducing the number of antigen presenting cells. It also promotes the release of immunosuppressive factors, induce suppressor t-cells, and decreases the ability of Langerhans cells to present antigen 94) A watt is a measurement of: A. PowerCorrect Choice B. Fluence C. Energy D. Distance E. Irradiance A watt is a measurement of power or irradiance of a UV source. Fluence and energy is measured in joules. These are related by the formula Joules/cm2=Watts/cm2xseconds 24
  • 25. 95) A patient presents with erythematous pruritic papules on exposed areas that appear in the spring. They appear between 2 hours and 2 days after exposure. Which of the following statements is NOT correct? A. The pathogenesis is unclear, but may be related to type IV hypersensitivity reactions B. It is more common in fair-skinned females C. Vesicles and eczematous dermatitis is uncommon D. The diagnosis described above is solar urticariaCorrect Choice E. This is an idiopathic disease that appears in the first three decades of life The diagnosis described above is polymorphous light eruption. The time to development of lesions is important in distinguishing between PMLE and solar urticaria. Solar urticaria usually develops 10-30 minutes after UVR exposure 96) Ultraviolet radiation from the sun causes all of the following acute effects in the skin EXCEPT: A. Photooxidation of preexisting melanin B. Redistribution of melanosomes from a perinuclear position into dendrites C. Epidermal thickening D. Mast cell degranulation E. None of these answers are correct Correct Choice All of these statements are acute effects of UV raditation on the skin. Immediate pigment darkening, which fades within minutes after exposure, is brought on by UVA and visible light. It is caused by photooxidation of preexisting melanin and a redistribution of melanosomes from a perinuclear position into dendrites. Epidermal thickening is mainly a UVB-induced phenomenon. Mast cell degranulation, with release of histamine and other mast cell products, also occurs as a result of UV radiation 97) The portion of the electromagnetic spectrum that produces a particular biologic effect is called the: A. Spectral Activity B. Cutaneous Effect Spectrum C. Action Spectrum Correct Choice D. Effective Spectrum E. Absorption Spectrum The action spectrum is the wavelengths that produce a certain biologic effect. The action spectrum for photosensitivity from exogenous chemicals is usually in the UVA range. The radiation that is absorbed by those chemicals is called their absorption spectrum 98) Which of the following genodermatoses is NOT worsened by sunlight? A. Darier's disease B. Kindler syndrome C. Rothmund-Thompson syndrome 25
  • 26. D. Hartnup disease E. Job syndromeCorrect Choice Many genodermatoses can be exacerbated by sunlight including Darier's, Kindler, Rothmund Thompson, Hartnup (pellagra changes 99) Actinic prurigo (AP) differs from PMLE in all of the following except: A. Cheilitis is rare in AP and common in PMLE Correct Choice B. The lesions of AP occur on all sun-exposed areas C. Outbreaks of AP are not as clearly related to sun exposure D. Lesions of AP may occur on non-sun exposed areas E. The lesions of AP begin in childhood Cheilitis is common in actinic prurigo and not a usual accompaniment in PMLE. 100) The MPD of Oxsoralen plus UVA is: A. Tested on the patient’s calf or abdomen B. Equal to one-half the patients MEDA C. None of these answers are correct D. Helpful in starting PUVA therapy Correct Choice E. Measured at 24 hours MPD stands for the minimal phototoxic dose. For Oxsoralen plus UVA, the MPD is measured at 48-72 hours. Testing is done on the upper buttock or forearm 101) The most common presentation of a patient with medication photosensitivity is: A. Lichenoid eruptions B. Photoonycholysis C. Fixed erythematous patch D. Pseudoporphyria E. Diffuse erythema in sun-exposed areasCorrect Choice Most patients with medication photosensitivity present with diffuse erythema in sun-exposed areas. In some patients, the eruption is eczematous and covered areas are spared. Photoonycholysis, lichenoid eruptions and pseudoporphyria do occur with mediation photosensitivity, but are not the most common presentation. Fixed erythematous patch is not seen with this type of reaction 102) Which of the following is the most common photodermatosis? A. Polymorphous light eruptionCorrect Choice B. Hydroa vacciniforme C. Solar urticaria D. Actinic prurigo E. Chronic actinic dermatitis 26
  • 27. Polymorphous light eruption is the most common photodermatosis. It is a idiopathic disease that usually appears in the first three decades of life and is more common in fair-skinned females. The pathogenesis is unclear, but is believed to be related to a type IV hypersensitivity reaction. Most lesions are erythematous pruritic papules, with the plaque form being less common. Lesions appear symetrically on exposed areas after a delay of several hours to several days. Patients with mild disease are treated with sun avoidance and a broad spectrum sunscreen. In more severe cases, hardening and desensitization can be accomplished or antimalarials can be used for resistant cases. For brief, sunny vacations, a short course of prednisone can be helpful. The other options are less common forms of idiopathic photosensitivity disorders 103) UVB is about how many times more erythermogenic than UVA ? A. 1000 Correct Choice B. 5 C. 10000 D. 10 E. 100 Though UVB is 1000 times more erythermogenic than UVA, UVA is much more plentiful in sunlight. Therefore, UVA does contribute to sunlight erythema 104) Phototoxicity and photoallergy from exogenous agents typically involve absorption of: A. UVB, UVA and visible light B. UVA Correct Choice C. UVB D. UVA and visible light E. UVB and UVA Topical and systemic agents that produce phototoxicity and/or photoallergy usually have action spectra in the UVA range 105) The UVC portion of the electromagnetic spectrum extends from: A. 10-200 nm B. 200-290 nmCorrect Choice C. 400-760 nm D. None of these answers are correct E. 290-320 nm Ultraviolet C does not reach the earth's surface. It is filtered out by the ozone layer. It extends from 200-290 nm 106) Ultraviolet radition has been shown to do all of the following in in vitro and in vivo studies EXCEPT: A. Increase circulating levels of IL-6 B. Suppress the induction of delayed-type hypersensitivity 27
  • 28. C. Decrease circulating levels of IL-1Correct Choice D. Induce suppressor T-cells E. Alter the ability of antigen-presenting cells to present antigen UV-irradiated mice have been shown to have defective antigen presentation and a decreased number of antigen-presenting cells, which prevents a normal delayed-type hypersensitivity response. UVR causes the release of immunosuppressive factors, with induction of suppressor T- cells and increases in circulating levels of cytokines, including IL-1, IL-6, and TNF 107) Narrowband UVB bulbs emit predominantly at: A. 290-320 nm B. 305 nm C. 360 nm D. 311 nm Correct Choice E. 352 nm Narrowband UVB (311-312 nm) is more effective than broadband UVB for psoriasis, vitiligo, and other skin disorders 108) Which of the following is not true about UVB radiation? A. Responsible for sunburn B. Decreased with high wind velocityCorrect Choice C. Virtually all blocked by car window glass D. Peaks at noon E. More intense in the summer than winter months The UVB band extends from 290 to 320 nm. The UVB spectrum is recognized as the primary cause of sunburn, skin cancer, and other harmful effects on human skin. The UVA band extends from 320 to 400 nm. This spectrum is further subdivided into UVA-2 (320 to 340 nm) and UVA-1 (340 to 400 nm). The UVA spectrum is recognized as a cause of immediate and delayed tanning reaction of skin, and several other effects including photoaging, skin photosensitization, and immunosuppression. UVC radiation comprises wavelengths shorter than 290 nm (from 200 to 290 nm). Notably, window glass filters out ultraviolet wavelengths shorter than 320 nm, so both UVB and UVC are effectively filtered by car window glass. UVB radiation is more intense during summer months compared to winter months and peaks during midday hours. It has been postulated that physical factors such as high temperature, high humidity, and wind can all increase susceptibility to UV-induced carcinogenesis 109) Which of the following statements about UVR in vivo is correct? A. decreases circulating levels of cytokins (IL-1, IL-6 and TNF) B. normalizes Langerhans cell morphology and function C. normalizes cell trafficking D. normalizes proportions of lymphocyte subtypes in peripheral blood E. induces skin cancersCorrect Choice UVR can induce skin cancer. UVR in vivo INCREASES circulating levels of cytokins (IL-1, IL-6 and TNF) and alters Langerhans cell morphology, cell trafficking, and the proportion of lymphocyte subtypes in peripheral blood 28
  • 29. 110) Which of the following is NOT true regarding polymorphous light eruption? A. It may occur through windowglass, which filters out UVB B. Usually appears in the first three decades C. Not all exposed areas show lesions D. May be a manifestation of a type IV hypersensitivity reaction E. Vesicles and an eczematous dermatitis are a common presentationCorrect Choice Polymorphous light eruption is the most common photodermatosis. It is an idiopathic disease that usually appears in the first three decades. Pathogenesis is unclear but it may be related to a type IV hypersensitivity reaction. Most lesions are erythematous, pruritic papules. The plaque form is less common, and vesicles and an eczematous dermatitis are rare. Not all exposed areas show lesions, but the same areas are affected year after year. It may improve as the summer progresses. It may occur through windowglass, which filters out UVB 111) What is the wavelength of a Wood's light? A. 365nmCorrect Choice B. 410nm C. 290nm D. 311nm E. 330nm A Wood's light emits ultaviolet light at a wavelenth of 365nm and is produce by bassing light through a Wood's filter which is composed of nickel oxide containing glass 112) The most common cause(s) of topical phototoxicity today in the United States is(are): A. 6-methyl-coumarin B. Musk ambrette C. PABA and non-PABA sunscreen ingredients D. PsoralensCorrect Choice E. Halogenated salicylanilides Topical phototoxicity is most commonly caused by psoralens. Topical 8-methoxypsoralen is used therapeutically to treated psoriasis, localized vitiligo, and hand/foot eczema. Psoralens in certain plants, fruits, and vegetables can produce phytophotodermatitis. Topical photoallergy has in the past been caused by halogenated salicylanilides, as well as musk ambrette and 6-methyl-coumarin in fragrances. These compounds produced photoallergic contact dermatitis, and have been removed from marketed products. PABA, its esters, and non-PABA sunscreen ingredients are the most common causes of topical photoallergy, not topical phototoxicity 113) Which of the following statements is true regarding ultraviolet carcinogenesis? A. UVA is most effective in producing pyrimidine dimers, which may activate oncogenes B. Cells from patients with actinic keratoses have normal DNA repair capacity C. Suppressor T-cells arise in UV-irradiated hosts only after tumors have developed 29
  • 30. D. UVA, when added to UVB, may accelerate carcinogenesisCorrect Choice E. Mid-range ultraviolet radiation is less efficient in inducing neoplasia in mice that is long wave UVR Mid-range UVR (280-320 nm) is more efficient in inducing neoplasia in mice, but long wave UVA, when added to UVB, may accelerate carcinogenesis. Suppressor T-cells induce susceptibility to tumors, and appear to arise in UV-irradiated hosts prior to tumors developing, thus playing a role in carcinogenesis. Cells from patients with AKs have less DNA repair capacity than controls. UVB is most effective in producing pyrimidine dimers, which may activate oncogenes, particularly in the formation of BCCs and SCCs 114) A patient presents with purple polygonal pruritic papules on sun exposed areas. Which of the following of his medication would not be suspect for causing this eruption? A. Hydrochlorothiazide B. Enalapril C. Quinidine D. FurosemideCorrect Choice E. Fenofibrate Furosemide (Lasix) is not a cause of lichenoid drug reactions, but can cause pseudoporphyria. The others listed are causes of lichenoid drug reactions. Others are antimalarials, demethylchlortetracycline and quinine 115) Which of the following drugs is commonly known to produce photosensitivity? A. Griseofulvin B. All of these answers are correctCorrect Choice C. Sulfonylureas D. Quinidine E. None of these answers are correct Quinidine, sulfonlyureas, and griseofulvin are all known to cause photosensitivity 116) The differential diagnosis of this patient would include: A. Chronic actinic dermatitis B. All of these answers are correct Correct Choice C. Airborne contact dermatitis D. Photoallergic contact dermatitis E. Drug photosensitivity NEEDS EXPLANATION 117) The action spectrum for photoallergy is mostly in which spectrum? A. 400-760nm B. 311-312nm C. 200-290nm 30
  • 31. D. 290-320nm E. 320-425nmCorrect Choice The action spectrum for photoallergic dermatoses refers to the specific wavelengths of light that evoke the photosensitive reaction. This falls mostly within the UVA region and may spill into the visible light region for photoallergy (320-425nm). 200-290nm refers to the UVC region; 290-320nm refers to the UVB region; 311-312nm refers to narrowband UVB region; and 400-769nm refers to the visible light region 118) This disease can be brought on by: A. UVA I B. All of these answers are correctCorrect Choice C. Visible light D. UVA II E. UVB Solar urticaria can be brought on by UVB or UVA or visible light or combinations of those wavelengths 119) All of the following are true regarding polymorphous light eruption except: A. Anti-Ro antibody positiveCorrect Choice B. Hardening occurs with subsequent episodes C. Abnormal metabolism of arachidonic acid D. Pruritic E. Lesions heal without scarring Polymorphous light eruption is the most common photodermatosis that is characterized clinically by the abnormal occurrence of pruritic, erythematous, edematous papules following exposure to UV radiation. Lesions heal without scarring. It tends to affect women 2-3x more than men. Positive Anti-ro antibodies should raise the suspicious for subacute cutaneous lupus erythematosus (tends also to be less pruritic 120) Most fluorescent UV sources are: A. Low pressure xenon arc lamps B. Low pressure mercury vapor lamps Correct Choice C. Low pressure argon lamps D. High pressure tungsten lamps E. High pressure xenon arc lamps The mercury vapor in the fluorescent bulbs is excited by electric current. Then the mercury emits radiation at 254 nm. This radiation is absorbed by the phosphor lining the bulb 121) Actinic prurigo (AP) differs from polymorphous light eruption (PMLE) in that: A. Lesions of PMLE occur on all sun-exposed areas B. Lesions of AP usually begin after puberty 31
  • 32. C. Lesions of AP may persist for months, even into the winterCorrect Choice D. Chelitis is more frequently seen in PMLE E. Lesions of PMLE may occur on non-sun-exposed areas Actinic prurigo (AP) may be a distinct entity, or an HLA-restricted subset of polymorphous light eruption (PMLE). AP differs from PMLE in that the lesions of AP always begin in childhood and often remit in puberty, the lesions of AP occur on all sun-exposed areas and may persist for months, even into the winter, and the lesions of AP may occur on non-sun-exposed areas. In addition, outbreaks of AP are not as clearly related to sun exposure, and chelitis is frequently seen in AP, not PMLE 122) This patient presented with hyperpigmented streaks after a vacation in the Caribbean. The most likely diagnosis is: A. Actinic prurigo B. CAD C. PMLE D. Phytophotodermatitis Correct Choice E. Melasma Phytophotodermatitis requires exposure to certain plants or fruits followed by sunlight. Parsnips, parsley, figs, limes, celery, bergamot oranges, and others contain psoralens that react with UVA. Initially there is erythema and blistering followed by streaked hyperpigmentation 123) Which of the following is the most likely cause of photosensitivity? A. Penicillin V B. Quinolones C. Sulfonamides D. DoxycyclineCorrect Choice E. Minocycline Doxycycline is the tetracycline derivative most likely to cause photosensitivity. Quinolones and sulfonamides will also cause this with ingestion. Minocycline is the least photosensitizing of the tetracycline derivatives. Penicillin is not a common cause of photosensitivity 124) Which of the following is true regarding immediate pigment darkening? A. Becomes prominent 48 hr after exposure B. Contributes to constitutive skin color C. Caused by UVA radiationCorrect Choice D. Requires the synthesis of new melanin E. Prominent in lightly pigmented individuals Tanning develops in two phases, early (transitory) and late (stable). The immediate darkening is in response to UVA and is related to photo-oxidation of pre-existing melanin 125) Which of the following statements is true regarding UV radiation, erythema, and pigmentation? 32
  • 33. A. Delayed tanning, which becomes visible about 72 hours after exposure, is largely brought on by UVA. B. UVB erythema reaches a maximum in 24-36 hours C. The chromophores involved with UVB erythema are melanosomes D. Immediate pigment darkening is brought on by UVA and visible lightCorrect Choice E. Immediate pigment darkening fades within 12-24 hours after exposure UVB in natural sunlight is the main contributor to erythema. UVB erythema reaches a maximum in 6-24 hours. The chromophores involved with UVB erythema are not clear but appear to involve nucleic acids. Immediate pigment darkening is brought on by UVA and visible light, and fades within minutes after exposure. Delayed tanning becomes visible about 72 hours after UVB exposure. UVA contributes to a lesser extent to delayed tanning 126) The main condition on the differential for polymorphous light eruption is lupus erythematosus. Which of the following tests should NOT be performed to help make this distinction? A. Anti-SSA B. Antinuclear antibody C. SED rateCorrect Choice D. Anti-SSB E. Skin biopsy for routine staining and direct immunofluorescence All of the listed tests are helpful in distinguishing between PMLE and lupus except a SED rate, which is a non-specific marker of systemic inflammation 127) As a result of ultraviolet radiation: A. Prostaglandins are increased B. None of these answers are correct C. There is mast cell degranulation and release of histamine D. Epidermal thickening occurs E. All of these answers are correctCorrect Choice Effects of ultraviolet radiation include mast cell degranulation with release of histamine and other mast cell products, increases in certain prostaglandins and interleukins, and epidermal thickening, which is mainly a UVB-induced phenomenon 128) Which spectrum of UV is responsible for the conversion of 7-dehydrocholesterol in the skin to pre-vitamin D3? A. 290-320 nmCorrect Choice B. 10-200 nm C. 340-400 nm D. 320-340 nm E. 200-290 nm Ultraviolet B with the spectrum of 290-320 nm is responsible for the conversion of 7- dehydrocholesterol in the skin to pre-vitamin D3 33
  • 34. 129) What range of ultraviolet radiation has been shown to be most efficient in inducing neoplasia in mice? A. 280-320nmCorrect Choice B. >760nm C. 400-760nm D. 340-400nm E. 320-340nm Midrange UVR 280-320nm (UVB range) has been shown to be most efficient in inducing neoplasia in mice. Long-wave UVA, when added to UVB may accelerate carcinogenesis 130) A normal MEDB on untanned Caucasian skin ranges from approximately: A. 20-70 mJ/cm2 Correct Choice B. 2-6 mJ/cm2 C. 140-200 mJ/cm2 D. 15-40 mJ/cm2 E. 70-140 mJ/cm2 The MEDB can vary from institution to institution. In one institution, it may range from 20-70 mJ/cm2. In another institution, it may range from 30-90 mJ/cm2 131) Phototoxicity from exogenous agents is characterized by all of the following except: A. There may be apoptotic cells histologically B. There is cross-reactivity to similar exogenous agents Correct Choice C. It appears as an exaggerated sunburn and heals with hyperpigmentation D. Occurs in most individuals given enough of the drug and enough UVR E. It can occur on first exposure to drug and UVR With photoallergy, not phototoxicity, there can be cross-reactivity among similar agents 132) A patient with a personal and family history of multiple fibrofolliculomas may have: A. Tuberous sclerosis B. Muir-Torre syndrome C. Gardner's syndrome D. Birt-Hogg-Dube syndromeCorrect Choice E. Basal cell nevus syndrome Fibrofolliculomas are small, benign, yellow or flesh colored papules that are usually inherited in an autosomal dominant fashion and have a predilection for the face, neck and upper trunk. Birt-Hogg- Dube syndrome is an autosomal dominant condition characterized by multiple fibrofolliculomas, trichodiscomas, acrochordons, collagenomas. This condition is important to recognize due to its association with renal cell carcinoma. In addition, lung cysts and bullous emphysema are also features of the condition with spontaneous pneumothorax being a potential complication. Muir-Torre syndrome is an autosomal dominant condition caused by a defect in hMSH2 gene. This condition is associated with sebaceous neoplasms including sebaceous carcinoma, sebaceous 34
  • 35. hyperplasia, sebaceous epithelioma, sebaceous adenoma as well as keratoacanthomas. These can be markers for underlying malignancy in this condition; notably adenocarcinoma of the colon, breast, urinary tract, lung and endometrium. Gardner syndrome is an autosomal dominant condition caused by a defect in the APC familial adenomatous polyposis gene in which patients have multiple hamartomatous polyps of the colon with a high rate of malignant transformation. Cutaneous clues to the diagnosis include multiple epidermoid cysts, fibromas, and desmoid tumors. Other manifestations include osteomas, supernumary teeth, and congential hypertrophy of the retinal pigment epithelium. Tuberous sclerosis, (TS),also known as Bourneville's syndrome is caused by defects in hamartin and tuberin, found on chromosome 9 and 16, respectively. Patients with TS are at increased risk of muliple neoplasms including retinal hamartomas, angiomyolipomas, and cardiac rhabdomyomas. Cutaneous manifestations include ash-leaf macules, shagreen patchs, café-au-lait macules, confetti macules, facial angiofibromas, and periungual fibromas. As the name implies, basal cell nevus syndrome is associated with multiple basal cell carcinomas. In addition palmoplantar pitting, multiple milia and epidermoid cysts are seen. It is autosomal dominantly inherited and caused by a defect in the patched gene 35